Você está na página 1de 200
University of Mumbai MCA Ser Discrete Mathematics Syllabus wef Academic Year 2007-2008 “Lectures: @ hoursiweek ET One Paper: 100 marks!3 hrs duration Term Work: 25 marks Term Works 25 mara | T Number system 7 1. Decimal nunber sytem Rh | 2 Binary number system | | | 3. Octalmamberysters | 4! Hexadecimal sumbersystem | 5. Binary arithmene { 2. Normal forms 3. Truth Tables | 4 Equivalence and Implications $. Theory of infarence for statement caleulus 6. Predicate ealeulus | 7. Rules of Logic | 8. Mathematical Induction & Quantifiers [3 ses Rela ns & Diaraph Review of set concepts i Relations and digraphs Properties of relations Equivalence relations ‘Compuier representation of relations and digreph “Manipulations of relations Partially Ordered Sets (Poses) ‘Recurrence Relations: 1 Tower of Hanoi 2. erations 3. Homogeneous linear equations with constant coefficients 4. Difference table and finite order difference 5. Line in a plane in general position 5 Groups and Applications | 2. Monoids, Semigroups | 2. Product and Quotiens of Algebraic Structures 3. [somerism, Homomorphism, Automorphisen | 4. Normal Dubgroups in - Codes and Group Codes Classification of Languages 1. Overview of Languages . 2. Representation of Regular Languages and Grammars 3. Finite State Machines =F \ References: 1 “Discrete Mathematical Structures": Tremblay and Manohar, Tata MeGrav Hill. 2. “Discrete Mathematics”: 1* edition by Maggard, Thomson 3. “Discrete Mathematics”: Semyour Lipschutz, Varsha Patil I™ Edition Schaum's Senes TMH 4 "Discrete Mothematiel Sicutures™ Kania, Rusby and R Hall India Edition 3 5. “Elements of Discrete Structures C. L. Liu ! 6 “Computer Fundamentals" P. K, Sinha f 7. "Discrete Mathematics and its Application”: Rosen 8. "Discrete Mathematical Structures": G. Shankar Rao, New Age ‘ 9. “Fundamental approach to Discrete Mathematics": AcharjayaD. P. Shivkuma, New f Age f- Guidance available for 1 [BSc | Maths, Stat, Discrete Maths, Numerical Methods | ! BSe(ITICS) Maths-LILII1, Discrete Maths, Numerical Methods, l Data Structures, Computer Graphics 1 | BPharm 75.Com. | Maths, Stat i MCA Sem [DM COA MCA Sem-I P&S, DSCC. | MCA Semi [OR DCN i MCA Sem-IV— [AT 7 i : MBAMMS Quantitative Techniques, Statistics \ ' rngineering | Diploma, Degree cw h min as Prof SG. Vays, Phone No, 9820031384 ‘9n69868 Example 1.18: Perform the following [Dec-09} )23.25)0= fi) (32)9= Chie (1101 1.00)-(01001.1 iv) soto-1011 =? i) 032 _ iil) (11011.00),-(01 001.1 Tovo=1011 = 100011110 vy) 1000110+ 1110=101 Example 1.19 (i) Conver the following Binary number into decimal and octal representation (L011. 110)2 (ii) Perform following Binary multiplication & binary Division: (1) 99010111%0000011 — (2)00101010=00000110 (ii) Perform following Binary subtraction: (00100101) ~(00010001) : (May-10} Solution: (1) (101 1.110} (ii) Q) 00010111*0000011 (Gi) (00100101) -(00010001 = 11.75)6 and (1011.110)>= (13.6 10000001000101 _(2)00101010+00000110=0111 Exercise 1: Perform the following, (Q)@31.25)i0= (> 2) (1302)= ie G)(1111.00)2-(0101.1)2=(?)s (@) Most =? (5) 1001110 1110-9 (6)(739.55)0= (M461= Oe (8) (11011.11)2(1101.01).=(7): (9) 101110*1110 =? (oy 1010+ 10110 =7 Prof S.G. Vaidya (M) 9820031384 2.Propositions and Logical Operations {Syllabus: Propositions and logical operations: Notation, Connections, Normal forms, Truth Tables, Equivalence and Implications, Theory of inference for statement calculus, Predicate caleulus, Rules of Logic, Mathematical Induction & Quamtitirs } 1 Propositions (or Statements) AA proposition (ar a statement) is @ declarative sentence that is either true or false, but not both matime, Consider following sentences in English (1) Mumbat is the capital of Maharashtra @) 12 {G) Do you play computer games (4) 37-51 (5) Bring me Valdya Sir’s book on Discrete Mathematics (6) No, thank you (7) The Sun will come out tomorrow (8) The square of 12 is 127 (9) Do you know C++? In above sentences 1,2.4,7.8 are statements in loyic but the other are n - 2 Connectiv tional 3 Logical Connectives . We can form new sentences from given sentences using ‘and’, ‘but’, “if ete. In the same way’ ‘we can get new propositions trom given propositions using ‘connectives Let us define following connectives Negation (SOT) Conjunction (AND) Disjunction (OR) Conditional or Implication (If. then ...) Equivalence or biconditional ( Ifand only if) 2 2.2.L.Negation (NOT: If P is a statement, the negation of P is the statement ‘NOT P'. In /mbols itis written as —P or ~P. Truth Table for Negation is given below : P| = z T IF @ Ft : 2. Conjunction (AND): If P and Q are two statements, then their conjunction is a Satemantaf the form P AND Q. In symbols it i writen a PQ i ce ‘Truth Table for Conjunction is given below F Prof SG. Vaya, Phone No. 9820031384 919369868 1. Number System [Syllabus: Number system, Decimal number system, Binary number system, Octal number system, Hexadecimal number system, Binary arithmetic} LA Number System A number system is defined as a set of values to represent numerical quantity. Common characteristics of all number systems are (i) Base or Radix, (i) The largest value of @ digit, and (Gi) Place value of «digit. Some common number systems used in practice and their bases are shown in the following table Name of the number system | Base (Radix) Decimal fo) Binary 2 Octal Hexadecimal 16 ‘The largest value of a digit is always less than the base. The place value of a digit is represented by different multiple of base 4.2 The Decimal Number System : It uses the base 10. The largest value of a digit is 9. Each place represents a different multiple of 10. The place values of each position are showm below. (Decimal number A T Place value Ie hee Fe oe i} 13 The Binary Number System: It uses the base 2. The largest value ofa digit is 1. Each digit in the binary system will assume only 2 different values 0 and 1. Each place represents @ different multiple of 2. These place values of each position shown below. [Binary number I : I Place value PoE ae .4 The Octal Number System It uses the base 8. The largest value ofa digit is 7 Each digit in the octal system will assume & different values from 0 t0 7 (0, 1,2,3.4, 5, 6, 7). Each place Tepresents a different multiple of 8. These place values of each position shown below. 1 (6-105, Nerul Station Complex CH), Fzst FLOor Prof. SG. Vaidyo, Phone No 9820031384 oewrenees [Octafnumber Place value = ee tee S The Hexadecimal Number Svstem: It uses the base 16 Each digit in the hexadecimal system will assume 16 different values, These values include digits 0 through 9 and letters A, B, ae A A i cy i CDF. Hexadecimal ] 0 PREEEEEZEP B lc {D Fy agi | | P= PPE EEE pepe papas ach piace in amid number represents « Uiieent multiple wf 1b These maulples ate sso caedas weighed values The weighted values of cach psion ae shown blow , Decimal number ie 1 1 Place value Tee Te hee ] I Example Ll: Conver (10101):-2)i0=h« (Ans: 21)0"(15)4) MS i Example 12: Conver (403) {Ans (259)0=(10000001 1} 1 Example 13 Convert (5C9)u=(?ha {Ans (1481) 9] ' Example 1.4: Convert (6AD)e=(?io=)s {Ans: (1709) (11010101101)3} \ Examule 13: Perfor he owing (May-08} : Ewes alas ies 101 =): Pa 1Q111«1010 = 199010 Zs on1010l + O101'=? (110001 .01 it) 43n= O50 tin) (1010),-(101);=(0101)- 1) 01-1010 = 1900 ¥) 0110101 + 0101 =1010.10 Example Lf Porform the following {Dee-07} 3 6-105, Ne Staton Compe (Wiest Floor ion: i) (45.625)0= (101 101.101). ii) (1011.1)2-(100.11)2=(0110 11) ili) (642):= 417)0 iv)0111*0101 = 100011 ¥) 0110101 + 0101 = 1010.10 Example 1.15: Perform the following )CIONLIO:= Cho ii) (413)e= Cho it) (1011.1) (100.11)2=O: iv) 1OM1*1010 =? v) 10100 + 101 Solutionsi){1 1011 110): (27 75h i) 413 = 26Te iit) (O11. 1-200 11),=(0110.11) iy) 10111010 = p01 ]10 v) 10100+ 100 =101 Example 116° Perform the following. (HON 110)>* Mio i) 213)e= Cho sii) (1101)z-(1001)2 =) ivy LOlt=1010 =? v) 10100~ 100 =? Solution: i) (11011.110)2= (27750 i) 213).= (139) ii) (1101)>-(1001 jp (0100), iv) 10111010 » 1101110 vy) 10100+ 100= 101 Example 1.17: Perform the following 1) (3625)i0 Oe ii) Q21)e= Oo vit) (1011.1 iy) Loon 11101 y) 10001111 = 1101 ‘Solution: i) (56.25)0= (10010001): ii) G21)e= (209):0 iit) (01011.11:-(01001.1Y5=(00010 01). iy) 10010*1101 =W110:11 v) 10001111 1101 = 1011 (01001.1)s=0) Piof $0" Vaidya, Phone No. 9820031388 s 9819369868 [Dee-08] [May-09) [May-11] 6-105, Neu Sion Compe (i), Fie FF figs lA (a3 \5 ~ Qeurak Dero gy = qeknsP se Ye ana gram Bae 6 = er: Example 1.18: Perform the following (Dec-05} 1) (23.25)10~ Os ii) (132)e= he ii) (1011 0}e(01001.)5=0 glove iv) 010«1011 =? 4 vy) 1000110 1110 =7 13-202 P iil) (11011 00).-(01001.1).=(10001 10): iv) 110101011 = 100011110 ¥) 1000110 1110=101 Example 1.19 (2) Convert the following Binary number into decimal and oc (Gor 110): (ii) Perform following Binary multiplication & binary Division, (2) 00010111*0000011(2)00101010=000001 10, (ii Perform following Binary subtraction: (00100101) -(00010001): representation (Miy-10} Solution: ())(1012.129}.= (11.750 and (OT1.L10= (13.65 {Gi)(1) 00010111*9000011=c00000001000101 (2) 00101010=00000116=0111 (iii) (00100101) 2-(0001000) Exercise 1: Perform the following a (C31 25)0= (2 2far 27281 (2) (1302). (?)1e ee 1 glist G)AI109,-(0101.):=02 4 4 (4) 10111 =? aes al ot 1 2 (5) 1001119 1n10= 4 (©) (739.55}0= Oe ee (1) 461 a= (Pho (8)(11011.11 (1101 01}24(%)2 2) BIneRaEO (@)lo1110«1110=7 3 eens (19) 1101110 10110 =7 a eS 1 = \ 1302 0.28 eu cee 2 ° 1 sae 1-6 Si ose ae jon ay y\voe 50105, Ne Station Comps, Fist Flos: Prof S.G. Vaidya (Mt) 9820031388 2.Propositions and Logical Operations [Syllabus: Propositions and logical operations: Notation, Connections, Normal forms, Truth Tables, Equivalence and Implications, Theory of inference for statement calculus, Predicate calculus, Rules of Logic, Mathematical Induction & Quantiers ] 1 Propositions (or Statements) A proposition (or a statement) is a declarative sentence that is either true or false, but not both Consider fallowing sentences in English (1) Mumbai is the capital of Maha hra 3) Da you play computer games (4) 3751 (5) Bring me Vaidya Sir’s book on Discrete Mathematics (6) No, thank you, (7) The Sun will come out tomorrow (8) The square of 12 is 127 F (9) Do you know C++? 78 statements in logle but the other are not e In above sentences 1,2, 2.2 Connectives (Propositional Connectivies or Logical Connec ‘We can form new sentences from given sentences using ‘and’, ‘bu’, we can get new propositions from given propositions using “connectives Let us define following connectives Negation (NOT) Conjunction (AND) Disjunction (OR) Conditional or Implication (If. then ..) Equivalence or biconditional (If and only if) if, ete. In the same way 2.2.L.Negation (NOT): If P is a statement, the negation of P is the statement ‘NOT P*. In symbols it is written as -4P or ~P ap =o? Truth Table for Negation ts yiven Gelo Pe TF Fars |r 2.2.2, Conjunetion (ANDi: {PP and Q are to Statements, thea their conju Statement of the form P AND Q” In symbols it is written as P>Q ‘Truth Table for Conjunction is given below Prof §.G. Vaidya. QM) 9920031384, A-(® “i za PAQ a oI aaa rrr 2.2.3. Disiunetion (OR). If P and Q are two statements, then their disjunction is statement of the form ‘P OR Q’. In symbols it is written as PLQ. ‘Truth Table for Disjunction is given below Po [ro tft ioje [a E(t |x etre r 22.4. Conditi IF P and Q are two Statements, then their conditional statement is a statement of the form “If P inen Q”. In symbols its writen as P40 ‘Truth Table for implication is given below Po TP59 T “(7 1r T OIF fF BT [a Foie fr | 22.4.1 Antecedent and Consequent: In the conditional statement P+ the statement P is called antecedent or hypothesis and the statement Q is called as consequent or conclusion 242 Different ways of writing a Conditional Statement: The conditional statement PQ can be written in any of the following forms, () LEP then Gi) Pimplies Q Git) Ponly if (iv) Q provided that P ) — QieP. (i) Pis sufficient condition tor Q (ii) Qis necessary condition for P 2.4.3 Converse, Inverse and Contrapositive of a Conditional Statement: If P-+Q is a conditional statement then @) QP iscalled its converse, vy | Sra is cates ns nverse Gil) QP is called its contrapositive Prof. SG. Vaidya (M) 9820031384 wation: Let: Today is Sunday Q It is aholiday ‘Then () P+Q means {ftoday is Sunday then its @ holiday Gi) Converse: QP is ~_ Iftoday is a holiday, then it is Sunday ii) Inverse: PQ is fic is not 8 Sunday then itis not a hi (Gv) Contrapositive: -P is Iftoday is not a holiday then it is not Sunday Biconditionsl Statement or Ei IEP and Q aro statements, then their biconditional statement is a statement of the form * P if and only if Q’, In symbols it is written as PeoQ, ‘Truth Table for biconditional statement is even below +e EFI ee \ ee LS Logica! Identities or Laws of Logic: Some equivalence are useful for deducing other equivalence. We call them identities or laws of logis Some fe _ esis s4st¢ 06m) Idem potent laws: (3) pap = p (b) pvp =P. Ws Comin dos ap (bhp.4 =a-P Assotiative laws: (a) (paq)ar=palaar) —_(b) (pvqyvr = pulqvr) 7 Distributive laws: (a) patque) =(paqyetpeny Cbypu(qas) = (pva)a(pes) De-Morgan’s laws:(a) s(paq) = —pv—1d (b) -tpva) 9. Identity laws: (a) pr’ 0 Anmiilation laws: (a) p-F = F 71 Inverse iaws (a) p94 12, Absorption laws. (a) pap 13. Property of biconditional 4. Contrapositive ofa conditional statement 3 Remark: Recall that , whenever we take negation (i) Alle Some efor every <> there exist, . Gi) Ore>and Prof S.G. Vaidya (M) 9820031384 FoGwnple 2.1: IP cepresents “This book is good” and Q represents “ This book is costly” ‘Write the following statements in symbolic form [Apr-o4] 4) This book is good and costly. b) This book is not good but costly } This book: is cheap but wood 4) This book is neither good nor costly. ©) IF this book is good then itis costly = — tutions) PQ” W)=PAQ) QAP) Pa gh R9Q Hsanple 2.2, Find the truth values of the following proposition IF3 are not an integer. then FT Trae is an imeger 1 are an inteye Solution: Let P and Q be “Sis an integer", “1/3 isan integer espectively The proposition QQ tie (as P is false and © is false), andthe statement (0) is face Cr pre and Q is false) Bere tt Fe te proposition “tis raining” be the proposition“ have tind’, and Fone it Bropeation “L wil gow a mover Tmedate ie eigen propositional forms BXo-05, Mas-06) (6) IFit isnot raining and {have time then Twill go to-a movie. (ny pa) (©) Itis raining and Iwill not go to.a movie. (6) es not raining, = (4) will not go 10 a movie (©) vit goto a movie only fi isnot raining Septlon. Let F be the proposition “itis raining”. Q be the proposiion“! have sime”, and R be the proposition “I will go to a movie” Then DPGrOR PR gy —p de Roop Example 2.4. 1°P. Q, R are propositions given in exampie 1.3, write the sentences iy/English CL ‘Feomesponding io pr-Proverk PI QPRARQ) (QR) R>—PnQ Solution: Proposition (a) can be expressed as “I will acto a movie if and only if it is mot raining and I have time ” i Pree ion(b) can be expressed as “I will go to-a movie and only if Lhgue time Proposition (c) can be expressed as “It is nat the case thet I have time 6 f will go to a movie.” Proposition (4) can be expressed as Iwill go to.a movie en fis rer rain ve have time Example 25: Let P, Q, R be the following statements [Dec.99, May-05) P: Idrive over 65 miles per hour a Qi get a speeding ticker R: Tam angry Prof S.G, Vaidya (Mt 9820001388 Write the following statements in terns of P, Q, R and the logial connectives. (i) I get a speeding ticket only if Idrive over 65 miles per hour {Gi do not drive over 65 miles per hour and I do not get speedina ticket yet | am angry {ii) lam angry if | drive over 65 miles per hour and get e speeding ticket Solution (i) QP Gi) (APAQ) AR il) (AQ) RK Example 2.6: Let p, ¢. and r be the following statements / I will study diserete structures 4g; 1 will go to a movie # Lamin good mood (2) Write the following statements in terms of p, q and r and logical connectives i) If am not in good mood, then T will go to a movie {ii) Lill go to a movie and I will study discrete structures (b) Write the English sentences corresponding to the following statements WCAeavp) Gap or Solution (ai) Sq. Gi) anp7 {) (IFT am notin good mood then I will not go to a movie or study discrete structures {iy L will go to a movie and not study discrete structure structres if and only if am in good mood Example 2.7: Let P,Q,R be the following statements P: Twill study Algebra Q Lusi go to 3 picnic R: Lam in good mood Write the following statements in terms of P, Q, R and logical connectives ip Lwill go to picnic only if will not study Algebra (i) If am not ina good mood, then I will go to picnic: Solution (aX) Q>-P (il) “RQ Example 2.8. Let P.Q.R be the following statements [Dee-02} You get an A in the subject Q: You do every exercise in this book. R. You get an A in this class Write the following statements in terms of P, Q, R and logical connectives ‘Also write its converse and contrapositive in English as well as in symbolic form. Starement is “You get an_A in this class of in the subjecy if you do every exercise in this pace & yO IEB Solution. Given statement in symbolic form : Q->(R“P) Is converse is: (PVR)>Q if you get an A in the subject or you get an A in this class then you do every exercise in this book Prof SG. Vaidva (vp 9820031388 Its Contrapositive is: = (PvR)-> —Q OR (-PaaR)}> -0 If you don't get an A in the subject and you don't get an A in this class then you don’t do every exercise in this book Example 2,9: Write negation of [May-06) 77 i) All men are faim. ii) All boys can run faster than all girls (Dec-07} ili) Some girls tre more intelligent than all boys [Dec-07] iv) Some men are animals v) All men take water and wine vi) IFit snows, then they da not drive the car [Nov-03] vii) “No teacher ise milionaire [Dec-02] Vil) Male © nit 2 Solution: Negation of the given statements i) Some met are not animals, Equivalently: There exist @ man, which is not an animal ii)) Some boys can not run faster than all gels, Equivalently: There exist a boy. who eannot nun faster than all witls it) All girls are not more intelligent than all boys Itis better to write negation as: No girl is more i iy) All men are non-aninals, Its better to write negation as No man is animal ¥) Some men da not take water or wine vi) It snows and they drive the car vil) There is a teacher who is not millionaire vill) Mala is good Example 2.10: Write converse inverse and contrapositive ofthe following Ali) If weather will be good then | will travel (ii) [today is Sunday then itis & holiday (iil) 16x=2 then x Solution (Converse: IF wil travel then weather will be good Inverse If weather will not be good then I wil not travel Contrapositive: IF will not travel then weather will not be good (Gi) Converse: If today is « holiday then it is Sunday Inverse If today is not Sunday then it is noi « holiday Contrapositive: Iftoday is nota holiday then itis not Sunday Gi) Converse. Ix=4 then ¥ Inverse - If x22 then Contrapositive IPx?=4 then x=? ; Prof S.G_ Vaidya (M0 9820031388 Example 2.11; Rewsite the following statement without using conditional () Ifyou work hard, you will succeed (ii) itis cold, he takes a blanket (Gi) If productivity increases, then wages rise ‘Solution: (i) Let P denotes you work hard Q denotes you will succeed. ‘Then the given statement in symbolic torm becomes: P->t) But P2Q = =PVQ, So the given statement becomes: You will not work (ii) In is not cold or he takes a blanket. (ii) Productivity does not increase or wages rise you will succeed Example 2.12 Write an equivalent formula PA'DER) which contains neither hicenditiana! ‘or conditional [May-05] Solution: Pa(QeoR) = Pa (Q4R)ARQ)) = Pa(-QvR)a(-RvQ) A © / 24 Functionally Complete Set of Connectives: &// Gazstion Define functionally complete set of comectives Give two examples [Dee-99, Nov-ve, Dee-08, May-10, Dec-10] Answers("We know that there are ofghese dy —> and <>, But some “The set containing minimum number of connectives which are sufficient to express any _ziven logical statemt in symbolic form is called as the functionally complete set of connectives, There are following 10 tunctionally complete set af connectives, ) (oy) is complete set of connectives ‘The can be expressed interms of pq = alana) =a Gpyna) The -> can be expressed in terms of... p->q=—pvq The ¢> can be expressed in terms of =v. =. peaq= (p>q)a(q—p) —pyaavp) star pg) eae {5} is a Functionally complete set of connectives (id) {4} is complete set of connectives The v can be expressed in terms of =n. pq. = (pve) 25 (-pa-4) ‘The — can be expressed in terms of =, ‘The «> can be expressed in terms of >, = (p54) lap) {5.0} isa functionally complete set of connectives ee Prof. S.; Vaidya (Mp 9820031388 2.5 Well Formed F Definition 2.1 A propositional variable is a symbol representing any proposition We note that usually a real variable is represented by the symbol x. This means x is not a real number but can take real value. Similarly, 2 propositional variable is not a proposition but can be replaced by a proposition. Definition 2.2 » A well formed formula (fi) is defined recursively as follows. (fp ise propositional variable then it isa wit (ii) Wer is wih, then —c isa wiP Gi) IF cand Bare well-formed formulas, then (cB), (AB).(a-9B), (ae>B) are well- formed formulas A string of symbols is a wT end only i itis obtained by finitely many applications of (iii). Note: A wif is not a proposition, but we substitute the proposition in place of propositional variable, we get a proposition, For example A(PVQ) Af QAR) is a wit (PAQ) GQ is a wit For the sake of convenience, we can refer to.a wif as a formula 2.5.1 Truth Table for 8 Weil-Formed Formula: If we replace the propositional variables in a formula a by propositions, we get @ proposition involving connectives. Ifa involves n propositional constants, we 2° possible combination of truth variables of proposition replacing the variables, Bxample 2.13: Obtsin the truth value for a=(PVQ) A( P-3Q) (QP) ‘Solution, The truth ‘abe for the given wis given below PTT no | P50 | ROAPSO | OP la tal P| rs v T ln etal Gaia) F F T |F F/T] 7 r T FOF Elrie r FE TIF Definition 2.3: A tautology or universally true formula is a well-formed formula, whose ruth value is always True Definition 2.8: A contradiction (or absu Definition 2.5: Two wil’ a and B in propositional variables P), Pa, ..,P, are logically equivalent if the formula ap is a tautology ty) is a wif whose truth value is always False Prof $.G. Vaidya QM) 9820031384 (Or Two wif are logically equivalent if they have identical truth values ‘The two statements c and f are equivalent is written as “a=p) Example 2.14: Show that following statement are equivalent © PQR)) =(PQv(PR)) Gi) SPQ) = (Pr=Q) Solution: (i) Let us prepare the truth table 3 QR | PQ)? re) P5QuP IR) fs 4344) i442 alo] manana als aan) 444 aan nana sall ead Hed ve aoa E eae asd L Since columns (5) and (8) are identical given statements are equivalent (ii) Let us prepare the truth table PT oT 9 TesQ [ssa [Pad TIN | - F the} rl or |r a ihe eee Be F Filt|r] iF F Since columns (5) and (6) are identical, given statements are equivalent Example 2.15 » Determine which of the form given below are tautology or contradiction, @ PQ) we Q) Gi) P(Q>R)) 6 = (PIQ)-HP-R)) ({May-07] (ili) (pq) a (qr)>(p>r) (May-10) (iv) (PaQ)eo(sRvQ) vP [Dec-99] (¥) P90} 0-5) this statement a tautology? {Dec-08, Dec-10] (9 QaP)Qa-P) {(Dee-09] (i) <@aP-Q) [¥0)-09} (vill) (P(Q>R)) (PQ) (PR) ([May-07) patina Se Saver rei : (i) Let us prepare the truth table Prof SG. Vaidya (v0 982005151 PT QT =0 [e507 Poo [P50 0 PQ i TI/T| EF] T F F tlhe} 7] F of F Flry Fr] F F Fle} 7|{ FE FE Since given statement is always false, isa contradiction (i) Let us prepare the truth table Let = (P-+(Q4R)) and Given statement becomes anB (P2Q)(P3R}) PTO [ROR fa PoQ_ [PaR J@s0> |e Tae || | P1oR) | or) | (Psa! I Ve i [eer Trt an 7 | T Peale tit]F] F F t | F F a ee T/ FIT T T F T T F | F tle]F|] 7 F F | F Te ate | F FLT/T i r T t T | F F FiTjFl F T Lr tof Gls me ata : Fle|r| + t t foo 1 rf F eo eee m ' ‘Since given statement is always false, it is a contradiction. Gid.Let ws prepare the tat able, Let a (p-54) 4 (@—1)-s(p-9r) PLQURY p59 Toor Toga G@on] por] @ T/T/T T T T 2} TT) Ble oe F Foi thelr] re | or F rr thee} fF | ot F F | ft F(t{r{ ot | ot r tT] a F[t{r] t | F F rT )4 Fle|r{ + | t tilt FLFIF T T T | gf ere Since given statement is always true, itis a taurology r Prof S. G. Vaidya (M) 9820051385 (is) Let us prepare the truth table (PaQ}¢9(-RvQ) vP P [| Q@ | | -R] Prd | RvO| GRO) _| (PAQOGRVQ) VP (ar ener re) nom T Tr Tih le Ler Ta T (ete pre cre i F TIFT F Tle! TT T F F [T T F F[ T T F a r F a a A my a (a a | Itisnether tautology nor eantnd (0) Let us prepare the tuth table (PLOTS 1-0 [P50 | s05-P [PaO ets0o5F) TitTtF{F{ tT r: T TIFT F [Tl Fase aera cre T T Eas or i Since given statement is always true, itis a tautology (i) Let us prepare the truth table PTO] —P [ow Tr Ei TF F[F Palo oan FlF] Its neither tautology nor contradiction. (si) Let us prepare the truth table = PAPL-Q) | PAPI rE E T E E T T T It is neither tautology nor contradiction, = Prof SG. Vala (M0 9824 (Vi) Let us prepare the ruth ible Let a= (P—1Q->R))-(P2Q)-HP-R)) QoR [P5@oR [P50 [POR [Pog POR) T | | ! | Since given statement is always true. its a tautology 2.16 Given the tuth vas of iT and those of R and S as P. find the Za value ofthe following 2) (PatQaR)) v(PLQ) (RWS) 8) (PQ) VR) v ((=PAQ) v=R) AS) (May-05) Solution: Given P and Q are T and Rand § are F a) Consider Pa(Q-R)) vCPvO) (RWS) (ZONT AR) ATT) MAEVE) vet = (TA) AT oF Ae a als am alo] manana als! aang eee cee iad aint G55 Ba aaaale Baaanaa4 pice ce a ‘Consider (>€PQ) VR) » ((PAQ) v=R) 98) (TAT) vo) v (TAT) uF) 0) (AT VT) v ((FaT) vT) AF) = (FD) v(t) AF) v (TAF) vF Example 2417 Giver the trth values of X and ¥ a5 true" and Z as false, find the truth values ofthe following —(XAYAZ)a(-X-—A¥AZ))A-V—AZZ)) (98) Solution: Given X and Y are Tand 2 isF OAV AZ)M=XAVAZ) NYAZAZ)) (TATA) -(T AF) AT AFF) 0 oe ae We “tey, Example 2.18, Without using truth table establish the following tautolozy 6) PQ) [Pr-Q\(Qn-P)) Gi) (PLQ) A-( =PAC-QV-R))) v -PA-Q) v (PAR) Sol 1) LHS—~(P5Q) =-{PQ) 4 (Q5P)] = APLQ) 9 -OvP)) 4-PUD) V~ (~QuP)] 3 PA-Q) v (Qx-P)] Prof S.G. Vaidya (6) 9820031381 [Dec-98} [Dec-07] Equivalence law Implication law Demorgan's aw Demargan’s law Hence ~(Pe>Q) <9{(Pa~Q)(Qa-P)] is a tautology By (PAD) amt =PA-Qu-R yyw Pat (PQ) 6 P.-Q-RY) (PQ) PAR) CI (PQ (QAR) v ~PAl~“ (PUQaR)) v -PA-Qv=R) = (PMQAR)) v ~[Pu(QAR)] Q) v PaoR) EPO) PRY Pe-Q) « (-Po-R) R) (PQ AQVAR)) v ~P(-Qv-R) Demorgan’s law Demiorgan's lass Distributive law Associative law dempotent law Demorgan’s law verse lay Example 2.19- Establish the following result without using truth tables (Use laws of logie to show the following equivalence) (Po (0-2 B) > P—A-QVR) <> (PAQIOR, (it) (PAQ)MPA=Q) = P (iil) PoQ.> P3(PAQ) (PR) (9) pagan) vigaru(pan) er (vi) P1QUR) = (P3Q(P3R)) (il) (PvO)(Qa(Rv=P)) € (PQ) 4-40 V=RAP)) (ill) €P2Q) > PMP YQ) = (PQ) Solution(iy LH buns. @0yo-0) =Pa(Qv-Q) {Implication lave Implication law Associative law Demorgan’s law Implication law Distributive law ® [Dec-07.May-09} (Nov-03, May-08] [ov-06, May-11] (Dee-10] {Apr-04, May.06} [Dec-98] _ P-+(PAQ) =P (PnQ) (PVP)A-PVOQ) () HS = PQ)A(RQ) = (=PVQ) (RO) (Q.-P) «Q-Ry = QVinPaoR) Qvi(PvR)) APR WQ ha (-PAC-ann)) wgat)vipar) = (Pagar) Manrju(par) -PA~q) vqvp) ar (—pa~q) v(qvp)) ar “(p¥a) “(qup)) ar 1) LHS=(P+Q)v(P4R)) =(=PVQ) PLR) PvoP)( QvR)) =5PU( QR) =P-(QYR) (vil) LHS= (PLQ)(Qa(RV-=P))) =MPVOMQAR QP) APO MOARWQa-P)] AAPA) A= OW AR) AG PA=Q) AQ’ =R) 9 —Q.P) =(PAAQ) Al(Qv-R) af -QvP) (P00) Af Qv(AR AP) Pook SG. Vaidya (Mp 9820031388 Inverse law ee Identity law Implication law Disteibutive law Inverse law Identity law Implication law Implication lav Comin Distributive law De-Morgan's law Commutative law Implication law Associative law Distributive law Distributive law Demorgan’s law Inverse law Identity law ‘Implication law Associative law entity law Implication law Distributive law Associative law Demorgans law Double negation Associative law Distributive law: 4 rol SG. Vaidya (M) 9820031388 iS (iil) PQ) > (PP VQ) < (PQ) PAQ) > (-PU=PVQ)) PaQ)] v (-Po(PVQ)) Implication law PQ) v OP u(=PvQ)) Double negation =PV(=PvQ)) ¥ (=PAQ) Commutative law [=Pv(=PvQ) v-P} « [=Pv(-PvQ) v Q) Commutative law = [Pv PLQv=P] 4 (aPv—PVOv Q] Associative law (PvP -PvOH 4 (-Pv-P)v(Qv ) Associative aw PvQ) A (=PQ)) Tdemotent law =PvQ) Idemotent law Lxample 2.20 Suppose thatthe value of P+ is ‘true What can be said about the value o =PaQePvQ? Solution: Given P+Q is true There are three cases Case (1) Pis T and QisT ' SPAQe>PLQ = —TATOs THT 1 Feo T =FoT ' ae ' Case (2) Pis Fand QisT A PaQesPOQ = 4FATes FUT Tato tT Tet =T Case (3) Pis Fand QisF APAQEPVQ= FaFes FF = TaFes F =FOF om i 1 i i i 4 1 ictal 218005 tre enh th lo Co) (pee | ‘Solution: Given PQ is true. There are two cases Case (1) pis T and qisT A(pvq) 2-9 = (TT) OT =) oF Case (2) pis Fand qis F (pvg) 9 = (PVE) OF Prof S. G. Vaidya ( 9820031385 A) eT ToT Example 2.22: Write an equivalent formula ~ PA(QeR) which contains neither biconditional nor conditional [May-08) Solution: PA(QOR) PAQR) ARQ} Equivalence law Pall-QuR) (RQ) Implivasion law PAQUR) “(-RQ) Associative la 2.6 Normal Form of a Well Formed Fort We want to determine whether given statement is a tautology or a contradiction, We can use the method of truth tables, which is simpler There is other method also. In that ‘method we reduce the statement to so called normal form We know that two formulas are equivalent if anc! only if they have same truth table. ‘The number of distinct truth values for formulas in P and Q is 2*. So the number of distinct truth tables is 2° Thus there are only 16 distinct (nc1equivalent) formulas, and any formula in P and Q is equivalent to one of these formulas Literal IF is @ propositional variable then either for P or —P are called as literals Note: We use Sum for disjunetion and Product for conjunction Elementary Sum and Elementary Product ° An elementary product is « product of literals. An elementary sum is a sum of literals, For example, PA-Q, QP, PaQ, 8))) (Dee-08} (P(QAR))siP-00) {Apr-05, May-08,.May-09] ‘Pa (P-+Q). 8) -PvQ)1P AQ) [May-06} ©) (P-Q)A(-PaQ) (Nov-06, May-11] D PAP 30) +0 096.9) [May a) Poi =P1010-5-R))) PYP—(QU=Qe-R))) Implication law PUPVQU—Qv=R))) Implication law = PvPVQVQv=R Associative law PvQvaQ.=R Edempotent iaw ‘Thus PvQv—Q v=R is disjunctive normal form of the given formula Solus Be PAR). PQ (Pa(QaR))u( PQ) Implication law Pa(aQu-R))u( -PVQ) De-Morgans law UPA QMPARYM(PYQ) Distributive law PaQ)v(Pa-R)y =PVQ Associative law ‘This isa disjunctive normal form of the given formula ) — PaP-+Q) 4s

Q)a(-PAQ) = (=PVQ) APAQ) Implication law =P AQ) 9 (=PVQ) ‘Commutative law =PAQP)ul =PAQAQ) Distributive law Prof. §.G_ Vaidya (Mf) 9520031384 (sPraPaQ)v( =PAQAQ) Associative law PAQ)APAQ) Idempotent laws Ins the required dat of given formula, ) :P40=-P.0 i Hence (Pa(P>Q)) [Pa{-PvQ)}+-Q (®s-P)APAQ)) 30 = [PP PAG = (PvP) 0 (PQ) (P00 =PvQ Its the required daf of given formula my epawn) Sep) AC-av—1) pat-av-r) = (pe=g)(pot) Is the required duf of given formula, For the same formula, we may get different disjunctive normal forms, For example (PaQaR)v( (PAQA=R) and PQ are disjunctive normal forms of PaQ. Se we introduce one for more normal form, called the principal digunctive normal form or sum of products ‘canonical form in the next definition. The advantages of constructing principal disjunctive ‘normal forms are For a given formula, its principal disjunctive normal form is unique. Two formulas are equivalent if and only if their principal disjunctive normal forms coincide ‘Min Term: A min term in n propositional variables Py, is either P; or =P, For example, the min terms in P and Pz are PyAPy, —PyAPs, PyA—I ‘number of min terms inn variables is 2" Principal Disjunctive Normal Form. A formula « i in principal disjunctive normal form if. isa sum of min terms. is QsnQs._.AQu, where each Q: APyoP: The Construction to Obtain Princig ctive Normal Form of a Step1: Obtain disjunctive normal form, Step2, Drop elementary products, which .re contradictions such as (PP) Step3:IfP; and —P, are missing in an elementary product a, replace a by (anP,)ulcr »—P) Step 4 Repeat step 3 until all elementary products are reduced 10 sum of min terms. Lise idempotent laws to avoid repetition of min terms ‘anonical sum-of-product form ie principal disjunctive normal form QAR) Example 2.24 Obtain of a)a=Pu(-Pa - rot S. G. Vaidya (M) 9820031388, b) ce = (4PL5 Q)-»(-aP AR) ©) @ whose truth table is given below =l=)5)45]5/5)5I5 SPI IS)=|55)5)5)5 a1 SERB Sle Solution a) ois already ind to introduce missing variables (s P= (PAQM( Pa-Q) = (PAQAR)V(PAQA=R) Ml PASQARYU( PAAQATR)) ‘Therefore, the canonical sum-of-produets form of eis (PeQaR)PaQnaR):( Pa-QeR) P= Q4-R) ( “Po=QaR) QrR in xis already a min term, Now, by =P QP AR) 2 PY Q) v (PAR) Implication law = (PAQ)v APAR) De-Morgan’s law ((PAQAR){PAQA=R))U((=PARAQ)(=PARA=O)) (PoQ°R) (PQ AR) (PAR Q)o(—PARA—Q) So, the principal disjunctive normal form of ais (PAQAR)PAQAR)V(PAQARV(=PA-QAR), ©) For a given formula a, the truth table is given inthe following table Pal=)=]= [555 5,~ i =/=)5/5 5/5 /)olo S/S )5)5)e S)=RISS) pale We have Tin the a column corresponding to the rows 1.4.5 and 8. The min term corresponding to the first row is PAQAR Similarly, the min terms corresponding to the rows 4.3 and 8 are PASQAuR, SPAQAR and R) ©) PR)A(P0) Solution: 2) PA(P+Q) =Pa(-PVQ) Implication law This isthe required conjunctive normal form of given formula b) Use PsQ=(P+Q)s(0-4P) (PLQ)}E(PAO} 1 PoQ) 5 (PaQ] - (PQ) > -evQ)) Equivalence law PLQNPrQ] » {= (PaQ) v (PvQ)] Implication law PQPVPQ.Q) [PQ ~P.Q) Distributive law = PLQVPIAPLOVQ) A (PV Q) v (APx-Q)} De-Mergan's law PVQVPINPVQVO) A [(PV-QvaP) 0 (=PV=QV—Q)] Distributive law (PL QUPIAPVQVQ) 4 (>PYQu =P). A (aPVAQVQ) Associative law This is @ conjunctive normal form of the given formula Prof S.G. Vaidya (M) 9820031384 2 QUPAQ)M=PAnQ) = QU{(PV=P) 4 =Q) Distributive law QuiPyP)] IQ v=] Distributive law (QvPv=P) A(Q v=9) Associative law vis the required conjunctive normal form of given formula Since each of the elementary sums is a tautology, the given formula is a tautology me (PV(Q>R)) (Pv(-QuR)) Implication law = PAHO) De-Morgan’s law = -PA(QAaR) De-Morgan’s law and double negation Hence the principal conjugate normal form of PQs PLO Peo = (P-2Q) (Q-9P) Hence (PR) (PQ) = (-PAR) -[(P4Q) (QP) [PAR] APY (PLR) APLQ) QV) Example 2.26 Obtain the principal disjunctive and conjunc a) (PV=Q)- (Peo) 8) (-Pv=O)> (PQ) 6) (Paq) w(-paaar) Solution: a) Principal Disjunctive normal Form (-Pv-Q)-+ (Per0) A (PAQ)-> (Pr-O) PnQ) v (Ps-Q) PaQ)y [ey MPs Q)} (PAQ) v (PA=Q) (> Ps Q) fo find the Principal Conjunctive Normal Form consider the remaining terms in Principal Disjunctive normal Form. These terms are: (— Ps —Q) ais (Po(QAoR)= Pv [AP) vRY (PQ) “(QP ) A =QvP)] OUR fe normal form of fXov-03} [May-05, Dec-0%] [May-10] Take the negation of this we get the required Principal Conjunctive normal Form Hence the Principal Conjunctive normal Form is = PQ) 'b) Principal Disjunctive normal Form (GPv=Q)> (PQ) (PAQ)-+ (PQ) PaQ) v (PQ) (PQ) « [PQ) A= P= QI} = (PaQ)y (PAQ) (> Pa Q) a Prof S.G. Vaidya QM) 9820031384 = (PAQ) > PAnQ) To find the Principal Conjunctive Normal Form consider the remaining terms in Principal Disjunctive normal Form. These terms are: (PAQ) (= PAQ) ‘Take the negation of ths we get the required Principal Conjunctive normal Ferm. Hence the Principal Conjunctive normel Form is = (PLO) 0 (Pv =0) ©) Principal Disjunctive normal Form (pag) vi-paqnr) this itselfisa disjunctive normal form [(paqns)y (paqa=r)ly (=pagesy (pages). (pagent) (prqne) This is principal dat dicate L Question Explain with suitable esanple (1) predicate (2) quantitier May-11 [May-11] A Predicatel 's predicates 2 statement conteining one or more variables) Propositio *roposition\* values are essigned to all the variable in a predicate, the resulting statement is 2 propos BL eg ‘x<5"isa predicate '3<3" is a proposition <5" is a proposition “There exists an x such that x5" is a proposition, “For all x, x<5 or x25 "isa predicate which is also a proposition Sanat Ti reassert ich Gite Abela’ Gin worl pbs queaiier vavubla sled Gainetaer Oe Mates eg, ‘There exists an x such that x<5' The variable x is bound by the quantifier there exists. ‘A predicate P that contains a variable x can be written as P(x). A predicate ean contain ‘more than one variable. Example 2.27. Write in symbols: 1) There exists an x such that x<4 [May- 10) 2) For every number» there is & number y such that y=x+ 3) There isa number y sich that, fr every number x, y=x+ 48) Every rational mimber is real umber 5) all men are moral 6) Some women are beautifl Prof §.G Vaidya (6 9820031384 7) No woman is beautiful as well as intelligent 8) For all x, x<4 or x>> (May-10) Solution: (1) Let P(x) be °x<4". Then the proposition can be written as 3 P(x) (2) Let P(xy) denote the predicate, ‘y=x+1". Then the proposition can be written as Yx Poy) (3) Let P(x) denote the predicate, ‘y-x+1", Then the proposition can be written as 3y Vx Posy) (4) Let r(x) denotes x isa rational number and R(x) denotes x isa real number ‘Then the given statement can be written as ‘x (r(x)->R(s)). (5) Let M(x) denotes x is a man and m(x) denotes x is mortal “Then the given statement can be written as +x (M(x)}>m(x)), (6) Let Ws) denotes x is a woman and Bx) denotes « heautifit ‘Thon the given statement can be written as 2x (W(s)- BIS) (7) Let W(x) denotes xis a woman, [(s) denotes x intelligent and B(x) denotes x is beautiful ‘Then the given statement can be written as 3x (W(x}- ~[I()AB()]) Exaprfle 228° What is universal and existential quantifier? Write English sentences corfesponding to each of the following ayPx P(X) b) 3x QIx) ‘Solution: The quantifier “for all’ is called as universal quantifier and the quantifier “there exists’ is called as existential quantifier Gi) All satisfy the propemty P(x) (ii) Some x satisfy the property Q(x), Exam Use Qoy for “is a rational number’ and Rix) for “x is @ real number ‘Trans)afe the following statements using the quantifiers [98] 2) All rational numbers are real numbers b) No rational numbers are rel numbers 6) Some rational numbers are rel numbers d) Some rational numbers are not real numbers. Solution: 2) ¥x (Q(x) > R(X) ) b) x (fx) > ROX) ) ¢) 3x (Q(x) > R(x)) 8) 3x (QW) >-RG)) Exhmple 2.30. Using predicates and quantifiers convert the following statements into mmbolic form. Give the universe of discawsse wherever necessary Also negate hem in symbolic form as wel asin Enelish a) Every student in this clas has studied calculus ') No professors are ignorant ‘Solution a) Universe of discourse is this class Prot $.G. Vaidya (QM) 9820081384 ret x) denote’ isa staden’ And Cl) dents * assed eeu. (Sts) -> Cts) [Negation is There i student who has not studied callus (S(8) +-€(s)) b) Let P(x) denote x isa professor’. And I(x) denotes “x has ignorant’. Yi (P(x) —>I(4)) ‘Negation is: There is a professor who is not ignorant, 3y (P(x) I(x) Eyample 2.31, Express the statement as a logical expression involving predicates, quantifiers Aith a universe of discourse consisting ofall students in your school and logical connectives. Every student in your school has « computer or nasa iriend who has a computer.” [Nov-04] Solution: C(x). “x has a computer | Fey) “cand y are trends" | ex (Cla) v 3y (C(y) FE.9) )) Example 2.32. Ps) vis. person ([May-05] oly Write the predicate *x is the father of the mother of y" ‘Solution: In order to symbolize the predicate, we name a person called 2 asthe mother of y Obviously we want to say that x ithe father ofz and zis mother of y 32 P(2) AMizy)aFlx.2) By 333. Let A(x) x has a white color, Bix) x is a polar bear, C(x) x is found in cold son, over the universe of animals. Express the following using quantifiers [Dec-10} 1) There exists a polar bear whose color is not white ii) Every polar bear that is found in cold region has @ white color Solution: (i) 3x (B(x) -~ 8(s)) Gi) &X [BG9 ACC9] > AG) Example 2.34 Let K(x). x is a wo wheeler, L(x). x is a scooter, M(x). x is manufactured by Bajaj. Express the following by quantifiers ([Dec-07} ) Every two wheeler is a scooter (i) There is @ two wheeler manufactured by Bajaj that is not @ scooter Solution (i) Vx K(x) > Loy Gi) 3x EKCy AMI} + = Lexy Ne 2.35. Let the universe of discourse be D=f0. 1. 2, 9} Let Q(cy) be the ane “xiymx-y" Determine the truth values of the following [Dee-99, Nov-06, Maye (08,Dee-08), QQ), (iy2y ox QW) Gil Vy 3 Any), Gv) 3x 6x2) (w) ¥x By QE) (4) ¥x YY Oey) Solution: (3) Q(x.) is "s+y=9-" Q(,1) means “I+1=1-1" i ¢ "2-0" which is impossible. Hence given statement is False Gi) 3y YX Q(xyy) means “there exists y such that for every x x= If we take y=0 the for every x °x=y=x-y" means “x+0=N-0", which is true i » Prof S.G. Vaidya (M4) 9820031385 Hence given statement is true (ii) ¥y 3x QCcy) means “for every y there is some x such that xty—>-y It is not possible to find even single x which will satisfy the property for every statement is False y. Hence given (iv) 3x (4,2) means “there is some x such that x#2=%-2" It is not possible to find even single x which will satisfy the property, Hence given statement is False (4) ¥x 3y Q(xy) means “for every x there is some y such that <* If.we take y=0 the property is true for every x Hence given statement is True every sand tor every y the property S~y=X-y 1817 twiy 2x 9y Qos) means It is not possible to find such x and y. Hence given statement is False Z of Inference for the Predicate Cal v L ‘Singular or Specific General or Non-Sy cific —— a g ¥ iv 4 singular singular Universal Existential affirmative negative General General ¥ J J Universal Universal Existential Existential affirmative negative affirmative negative tag all roses are eg Nocirdles are eg. Some girls, eg. Some snakes set cmeing| squares re saan sre net poisonous 8 Rules of Inference For Prof SG. Vaidya (4) 9820031388 1 Cateut {In logic some propositions are assumed to be true. They are called as premises. Based on thet assumption some other propositions are derived (deduced or inferred). Those derived Propositions are called as conclusions Some important rules of logical reasoning or rules of inference are given below LAdgition: 3) Simplification 4) Modus Bones 5) Modus Tollens © Disjunctive Sullogisn ‘D Hypothetical Syllogism., 8) Constructive Dilemma 9 Destructive Dilemma P39 OR POR PQ AR45) PR os (P3Q) ARS) Prof $. G. Vaidya (M) 9820031384 Hypothesis or Premise: The propositions that are assumed to be true are called hypotheses or premises. Conclusion : The proposition derived by using the rules of inference is called as conclusion, Valid Argument » The process of deriving conclusions based on the assumption of premises is called a valid argument Exafaple 2.36 : Can we conclude S from the Following premises? [Mays06, May-11) (i) PQ Gi) POR Gi) QAR) (iv) SvP Solution | PQ) premise tip 2 POR, premise (ii) 3. (P3Q\(P3R) Tine 1,2 and conjunction 4 premise (ii) 5 line 4 and De-Morgan’s laws 6 line 3,5 and DD, io 1 Idempotnnt law 8,SP premise (iv) oS lines 7,8 and D.S : ‘Thus we can conclude S from the given premises Example2,37| Derive $ from the following premises using a valid argument [Apr-og, Dec-09} P40 QIK Gil) Be GR Solution 1. PQ premise () 2. Qo premise (i) 3 POR line 1,2 and HS aR premise (iv) 5.=CR) line 4 and double negation 6 line 3,5 and MT TPs premise (i) 8s lines 6,7 and DS ‘Thus we can have derived S from the given premises Example 2.38 : Derive the conclusion, using ules of inference if necessary ([May-03] =Qv RRS, “PvO =P» S - Solution: —QvR AQ R QoR ROS Qos =PvQ A-P)30 PQ Pos Prof SG. Vaidya (4) 9820031584 premise () line 1 and implication taw fine 2 and double negation premise (i) line 3,4 and HS premise (i) line 6 and implication law lines 7 and double negation line , 8 and 123 ‘Thus we can have derived S from the given premises. Example 2.39 Show that R->S can be derived trom the premises P-(Q98), “RP and Q [May-05, May-10} ‘OR. Show that the D-»€ is a valid conclusion for the premises AB30), =DvA, Solution 1. P-(QS) =RVP ROP R4(Q>8) ROS) = Ry (-QvS) Q aRvS Ros B {Dec-99} premise (i) premise (i) line 2, implication line 13 and HS line 4 and implication line $ and implication premise (i) lines 6,7 and DS. line 8, and implication. Example 2.40. Show that the conclusion D follows from the premises (ASB) MAIO), > (Br0), Solution!) (AB) (AC) 2) = (BAC) 3) (4Bv=C) 4) AVA. S)aA 6) Dva nD Hence the given argument is valid DvA [Dec.99, May-07] premise (i) premise (i) Tine 2, De-Morgan’s law line 13, DD. line 4, 1dempotent law. premise (i) line 5,6 DS. Example 241. Using the nals of inference show that SvR i tautology implied by (Pv Q) n(P->R) x(Q98) [Nov.04] Solution!) Pv.Q premise () 2)4P3Q line 1, implication 3) QS premise (ii) 4) PS Tine 1,3, HS. 5) 3S? line 4, contrapositive, O)PaR premise (i) 7) “SOR line 5,6 HS 8)SvR line 7, Implication Example 2.42 : Check the validity ofthe following arguments, If Ram has completed M.C.A or M.B.A, then he is assured a good job. If Ram is assured a ‘2004 job , he is happy. Ram is not happy. So Ram has not completed MC A. (Nov-5. Dect Let P denotes ‘Ram has completed M.C A. ‘O denotes “Ram has completed MBA denotes “Ram he is assured a good job 'S denotes ‘Ram is happy The given premises are (@PVQIR Gi) ROS i) 8 ‘The conclusion is —P 1 (PVQ)>9R premise (i) 2R9S premise (ii) 3. (P.Qhos line 1,2 and HS 4,98 premise (i) 5. (PQ) line 3, 4 and M-T. 6. PAnQ De-Morgan’s law 1 line 6 and simplification Hence given statement is valid Example 2.43: Test the validity of the following arguments [If milk is black then every crow is white. If every crow is white then it has four legs. If every crow has four legs then every butfalo is white and brisk. The mulk is birck. So every buffalo is white [Nov-06,May-09,May-10] Solution, « Let P denotes "The milk is black Q denotes “Every crow is white.” R denotes “Every crow has four legs S denotes ‘Every butfalo is white” T denotes ‘Every buffalo is brisk “The given premises are P30 Gi) QR GigR9SAT Prof S.G. Vaidya (Mp 9820031384 Salution'1, “Qv R premise (i) 2, (-Q)>R line 1 and. implication law 3.Q0R line 2 and double negation 4.ROS premise (i) 5. QoS line 3, 4 and HLS. 6. PQ premise (il) 7 =(-P)}90 line 6 and implication law 8 PQ lines 7 and double negation 5. Pas line 5, 8 and 1.3 ‘Thus we can have derived S from the given premises. Example 2.39 : Show that RS can be derived from the premises P(Q8). RP and Q [May-05, May-10] (OR Show that the D-sC isa valid conclusion forthe premises ABC), “DvA, BB [Dee-99} Solution: 1. PQS) premise () 2, 8) line 1,3 and H.S 5. R-i(-QV8) line 4 and implication 6 = RV (05) line $ and implication ql premise (ii) 8. sRvS lines 6,7 and DS ROS line 8, and implication Example 2.40. Show thatthe conclusion D follows from the premises (ASB) MAC), (BC), Dv [Dec-99, May-07] Solution!) (A+B) (AC) premise (i) 2) > Bac) premise (i) 3) (=Bv=C) line 2, De-Morgan’s law 4) AVA, line 1,3, DD. 5)nA line 4, 1dempotent law 6) DvA premise (ii) nD line 5,6 D.S, Hence the given argument is valid Example 2.41. Using the rules of inference show that SvR is a tautology implied by (Pv Q) APR) (QS) [Nov-04] Solution) Pv Q premise (i) ote ss Prof S.G. Vaidya (M) 9820031384 2) P20 line 1, implication 3) Qs premise (ii). 4)PS line 1,3, HS. 5) SP line 4, contrapositve PR premise (i) 7) 3S3R line 5,6 HS. 8)SvR line 7, Implication Example 2.42 ; Check the validity ofthe following arguments If Ram has completed M.C.A or M.B.A. then he is assured 2 good job. If Ram is assured a .g00d job , he is happy. Ram is not happy. So Ram has not completed MCA. [Nov-0S Dec.08] Solution. : Let P denotes ‘Ram has completed M.C.A, 10 denotes “Ram has complerot BA R denotes “Ram he is assured a good job 'S denotes “Ram is happy.’ The given premises are ()PVQSR GiyRoS i) -8 ©The conclusion is ~P LPvQ)9R premise (i) 2 RS premise (i) 3.(PvQp9S line 1,2 and HS 4-8 premise (i) SPQ) line 3, 4 and MT. 6. PQ De-Morgan’s law 7-P line 6 and simplification Hence given statement is vali Example 2.43 : Test the validity of the following arguments If milk is black then every crow is white If every crow is white then it has four legs. Ifevery ‘crow has four legs then every bulfalo is white .nd brisk. The milk is black. So every butfalo is white [Nov-06, May-09,May-10] Solution. Let P denotes “The milk is black" Q denotes ‘Every crow is white." R denotes “Every crow has four legs.” S denotes ‘Every buffalo is white T denotes “Every buffalo is brisk ‘The given premises are (P30 Gi) QR GiDRoSAT Solution’, -QvR 2. {-Q)>R Prof S.G. Vaidya (M) 9820031384 premise (i) Fine 1 and implication law line 2 and double negation premise i) line 3, 4 and HS premise (ii) line 6 and iemplication law lines 7 and double negation line 5, 8 and 5 Thus we can have derived S from the given premises Example 2,39 - Show that R-+ can be detived from the premuses PQS), “RP and May-05, May-10} OR Show that the D->C is a valid conclusion for the premises ADBIO), = DvA, Solution: 1. P(QS) ARVP ROP R(Q35) R(Q¥8) AR (-QvS) ARvS ROS B [Dec.99} premise (i) premise (ii) line 2, implication, line 1,3 and H.S. line 4 and implication line 5 and implication. premise (ii) lines 6,7 and DS. line 8, and implication Example 2.40 Show that the conclusion D follows from the premises (AB) NAO), (BAC), vA (Dec-9, May-07) Solution) (AB) (AC) premise 2) = (BC) premise () 3) Bv-c) line 2, De-Morgan’s law 4) sAvA line 1,3, D 5) =A line 4, Fdempotent lw 6) Dva premise (i) nD line 5,6 5 Hence the given argument is valid Example 241 Using the rules of inference show that SR isa tautoloy implied by (Pv Q) A(PR) (Q8) [Nov-04] Solution!) Pv. Q premise (i) Prof SG. Vaidya (Mt 9820031384 2)3P30 line 1, implication 3)Q8 premise (ii. 4) Po S line 13, HS 5) “SP line 4, contrapositive ©) POR premise (i) 7) -S3R line 5,6 HS 8) SVR line 7, Implication. Example 2.42 : Check the validity of the following arguments, If Rem has completed M.C.A or M.B.A, then he is assured a good job. If Ram is assured @ ‘2004 job , he is happy. Ram is not happy, So Ram has not completed M.C.A._[Nov-05.Dee-8) Solution. . Let P denotes ‘Ram has completed MCA. O denotes ‘Ram has completes MBA R denotes “Ram he is assured a good job S$ denotes “Ram is happy” The given premises are PVQSR Gi) ROS (ii) “8 ‘The conclusion is -P 1 PLQR premise (i) DROS premise (ii) 3 PQHs line 1,2 and 11S. 4.78 premise (i) 5. -(PvQ) line 3, 4 and MT 6 PanQ De-Morgan’s law 1? line 6 and simplification Hence given statement is valid, Example 2.43 : Test the validity ofthe following arguments IF milk is black then every crow is white If every crow is white then it has four legs. If every ‘crow has four legs then every buffalo is white and brisk. The mulk is black. So every butfalo is white [Nov-06,May-09,.May-10] Solution.» Let P denotes "The milk is black Q denotes “Every crow is white” R denotes ‘Every crow has four legs." S denotes ‘Every buffalo is white T denotes “Every buffalo is brisk * The given premises are (P50 Gi) QR GRSA7 Prot §.G. Vaidya (M4) 9820051384 (wP The conclusion is § 1.P3Q premise (i) 2.Q0R premise (i) 3. POR Line 1.2.8 4. ROSAT premise (i) 5. PoSaT Line 3,4 HS 6p premise (iv) 7.SAT Line 5,6 MP 8S Line 7 Simplification Thus the argument is valid Example 2.44 ‘If there was a ball game then travelling was difficult If they arrived on time then travelling was not difficult They arrived ot Show that this statement constitute a valid argument Solution, : Let P denotes ‘There was a ball game Q denotes ‘Travelling was difficult R denotes ‘They arrived on time The given premises are (PQ Gy R> = con The conclusion is —P 1R+-Q premise (i) 2k premise (ii) 3.39 Line 1, 2, MP. 4.P3Q premise (i) SP Line 3,4 M.T. ‘Thus the given statement constitute a valid argument Example 2.45 _ Determine the validity ofthe following ergument using deduction method, “IFT study then I will pass examination. If I do not go to picnic, then I will study. But I failed examination. Therefore, 1 went to picnic. [Dec-i0} Solution. - Let P denotes ‘I study * Q denotes ‘I will pass examination * R denotes ‘Igo to pienie The given premises are (P+ (i) RP (it) = Prof. G. Vaidya (M) 9820031384 c The conclusion is R LaRSP premise (i) 2P4 premise () 3. -R+Q Line 1, 2, HS 4.50 premise (i) 5. =CR) Line 3,4M.T oR Line §; double negation ‘Thus the argument is valid Example 2.46 : Consider the following argument and determine whether itis valid or not. Either I will get good marks or I will not graduate. If I did not graduate I will go to Canada. 1 get good marks ‘Thus, T would not eo to Canada [Dec-07] Solution. © Let P denotes ‘will get good marks ‘Q denotes I will graduate } R denotes ‘I will go to Canada ‘The given premises are (P= (i) ~QOR (ii) P ‘The conclusion is ~R. 1 Pv-Q premise (i) double negation Line 2, Implication law premise (ii) Line 3,4 HS premise (ii) 8. Conlusion may be R or ~R Line 7 Simplification ‘Thus the argument is not valid Example 2.47: Translate into symbolic form and test the validity of the arguments. [May-08} TFT play football, I cannot study. Either I study or I pass DBMS I played football ‘Therefore, I passed DBMS. Solution: Let P denotes’ ‘I play football Qdenotes: ‘I ean study.” R denotes: I pass DBMS. Given premises are @ P>-9 ; Gi) QvR Gi) P Prof S. G. Vaidya. (Mp 9820081388 ‘The conclusion is R Proof: 1)P->-Q premise (i) 2P premise (ii) 3)-0 line 1,2 M.P QR premise (i) 5)R line 3,4 D.S ‘Thus the argument is valid Example 2.48: Investigate the validity of the followi~z argument IFT go to my class tomorrow then I must get up early, and if | go to dance tonight I will stay Up late. If stay up late and get up early, then I will be forced to exist on only five hours of sleep. I simply cannot exist on only five hours of sleep. Therefore I must either miss my class tomorrow or not 20 te the di Solution: Let P- I yo to my class tomorrows QL must get up eatly RL yo to dance tonight, ‘S:will stay up late R: Twill be forced to exist on on only five hours of sleep. Given premises are @) Pa Gi) ROS Gi) (SAQR ()-R The conclusion is -PU-R Proof 1) -R premise (i) 2)SAQIR premise (i) 3) ($a) line 1,2 M.T. 4)-S/-Q line 3, De-Morgan’s law 5)P3Q premise i) ROS premise (i) 7) (PQ) ARS) line 5,6 conjunction 8) -PVAR line 4,7, DD. Hence the given argument is valid Example 2.49; Is the following argument valid? Justify [Dec-99} IFT try hard and T have talent then | will become a musician, If vecome a musician then I will bbe happy. Therefore I will not he happy then either I did not try hard or 1 do not have talent ‘Solution. Let P:Ltry hard Q Thave talent RI will become musician S Iwill be happy Prof $.G Vaidya (M0) 9820031384 Given premises are @ — PrQ>R (i) ROS. ‘The conclusion is -S(—-Pv-Q) Proof 1) PxQ) +R premise (i) DRS premise (ii) ; 3) (PaQ) +S fine 1,2 HS) 4) PaQuy § line 3, Implication law 5) Sv-(PAQ) line 4, commutative lav 6) =(-$):—(PQ) __ line §, double negation 7) (98) PO) line 6, De-Morgan’s law 8) -$-9(-PvQ) line 7, Implication law Hence the given argument is valid ‘Mathematical Inductio Ze P(n) denotes ‘the property is true for every 1. The following method of proving P(n) is called as the method of mit ematical induction or simply the method of induction Step 1» Show that P(1) is true Step 2. Assume P(k) to true for some k Prove that Pk+1) is true Consider 1b 243+-- 4k eke Ihe k=) H Ak +1420 2 _ + DE +2 _enjesien P+!) is true. Hence P(n) is true for every a, Solution: 3) Let P(n) be the property “2=4ro+ ... +2n= (nly for all n21 ‘Step: To show that P(1) is true Prof § G. Vaidya (M) 9820031384 LHS-2 RHS=1(1+1)°2 LHS-RHS. (1) is true ‘Step2: Assume P(K) to true for some ken, 24464. #2k = k(kt1) ‘Step 3: Prove that P(k+1) is tue Consider 2+4+6H,.,,, +2k+2(k+1) = K(k+1)#20+1) (e+ )K*2) P{k*1) is true, Hence P(n) is true for every 2. _ (ns Qn) Bayecem e Solution: 4) Let P(n) be the property “1 Step: To show that P(1) is true. Liis=1?-1 paisa eu 6 6 LHS-RHS PCi)istne oot ceca? = HHDQ sien Assume Pa) to eforsomekeen, 2 aatas? ea « Mees D Step 3: Prove that POI) is true Consider (Ui + =e (heel MEE) Senay He 2k +1) = 6k = 1 P(k+1) is true Hence P(a)is true for every wns)" Solution: 5) Let P(n) be the property 1° +2) +3° 40-477 ~ Step To show that P(I) is true LHS=1"=" (ne RHS~ LHS-RHS Piipis true RD Step 2° Assume P(k) to true for some k0 Hence 10k*10<34+3%<3"" for kod ie 10k+ 103" P(k+I) is true Hence P(n)is true for every no Solution: 13) Let Pn) be the propery 2"n! for n=4. ‘Step 1: To show that P(4) is true s Prof $.G_Vaidya (Mp 9820031384 For n=4, the statement 2" (PVR) = (RVQ) (3) Find the truth values of the component statements, given that 2) Pa-R)-H(Qa(P-08)) —9R) is tre (Dee-02} b) PnQ)> (P(Q>R)vS) is true [Dec-99} ©) If p-oq is false, determine the truth value of (~paq)-oq Explain your answer INov-03] (4) Establish the following result without usiig truth tables (Use laws of logic 19 show the following) (APLC) 3 (As) 40 Gi) -(PAQ) and sPV-0 (May-03] > [Dec-99] (S) Let the universe of discourse contain 9 elements namely the natural numbers 1,2, ....9 Let D stands for ‘divides’ and D(x.y) is true ifx divides y and false otherwise. Determine the value ‘true’ or false” for the following. 8) (Gx) (Vy) DEY) b) 9 (ry) Diy.n)) (5) IF the universe of discourse is the set {a, b, c), eliminate the quantifiers in the following Formulas P(x), (sx) Ris) AGS) Stx), (8) (PES) > Gy) QY) Prof S.G. Vatdva_(M) 9820031384 (PLerPCab,) be the statement “a+b=c", What are the truth values of the statements Be Va vb Pla.be)and Wa 3b 3c ‘Pla,b.c). [May-03] (8) Show that (3x) M(x) follows logically from the premises(x) (H(x)2M(x)) and (3x) H(s) [Nov-03] (9) Construct a formal derivation of the following argument Every member of the committee is wealthy and Republican Some committee members are old ‘Therefore there are old Republican (10) Using the method of induction prove that (i) Leet +e (20-1) = Fn Wfan+) 3 is divisible by 4. n=N We 2g Mice 13°33) 7 Gres) 2a) (i) san? is divisible by 3 for n= Bre tYon=7) (ii) 3°-20-1 is divisible by 4. neN 1 a “Gee Ga) 7 "is divisible by 25 for al ne (i) (cos@ +1sin 8)" =cosn0~isinn®. _neN 0 (vi) 134 244-4 nb wxina-|) a} oP ba | (sii) Se10+15+ _. +5n = for all nz [May-10] 5 1 mere Prof SG. Vaidya (8) 9820091384 3. Relations and Digraphs {Solabus’ Sets, Relations & Digraph: Review of set concepts, Relations and digraphs, Properties of relations, Equivalence relations. Computer representation of relations wal Aigraphs, Manipulations of relations, Partially Ordered Sets (Posets) | 3.1 Sett A collection of well detined objects is called as a set and the objects are called as elements of the set ) im general set is denoted by capital letters and elements are denoted by small letters 322 Partition of a Set: A partition of a nonempty set A isa collection P of its non-empty subsets A, such that Each element of A belongs to one of the sets in P TEA) and Ap are distinet elements of P, then AiAs=@. 3.3 Cartesian Product of ewo sets: {EA and B are two non-emply sets, their Cartesian product A*B as the set of all ordered pairs (0) with ac ang beB Thus A*B={(a,byacA, beB} “Note tha, for any finite, non empty sets A and B, |A-BI-[AlB) Any subset of AxB is called as relation from A to B We will use letter ike R, S ete to denote the relation 34 Sets_Arising from Relation’ Let RAB be a relation from A 10 B. We now define ‘asious important and useful sets related to R v ) Ao 70 The Daman of, dented by Don) saat of ements ae i (hyd Jo “Ci) The Range of R, denoted by Raa R),isaset ofelements beA if(a,byeRe>/ (Gi) _ TER isa relation from A to B and xe, we define R(x), the Relative set of x, © be the set ofall yin B with tne propery that x is Related 10 y Thus Ris)e(ve] (3) €R} Cy) _ Similarly if ACA, then R(A,), the R-reative set of Ay, to be the set of all ye with ‘the property that x is R-related to y for some x in A; That is. RCP tyeBl( IR since 1R2 and 2R4 since 2R2 and 2R2 since 2R2 and 2RS °S since 3R4 and ARS ince 1R2 an since 1R2 and 2 since 2R2 and 2R4 since 2RS and)SR6, 416 since ARS ang SRO Example 3.11) Let A=(ab.c4.e) and R={(@.9)(8,0).(6.0)(c.e)(€.4),(4.)). Draw digraph of R Compute aR’, b)R* {May-05] ‘Solution: The diagraph of Ris shown below. at cde, 7000 lOO Ve0 Ooolk 6 900 ©0000 OXF 0655 31 Prof SG. Vaidya (M) 9820031384 NS eo 4) To compute R® aR’asinceaRa and aRa aR’ since aRa and ab aR“e since aRb and bRe bR’e since bRe and cCRe BR’ since bRe and cRd. ce since cRd and dRe Hence R°=(a.ab, (a.b)4a.0). (b.eb. (B.A). 6.2} b) To compute R~ we need all ordered pairs of vertices for which there isa path of any length from the first vertex to the second. From the above figure we see that Re=H(a.a), (AD) AC), (2d), (Be), (B.C), (B.A), (B.e}, (EA), (6.6), (85 43.8 Properties of Relation 3.8.1 Reflective and Irreflective Relations ‘A relation R on a set A is reflective if (a.a)eR for all acA. (je Every element is related 10 itself) ‘A relation Ron a set A is jmeflective if (a,a)eR for every ac A. (je No element is related to itselt) 3.8.2 Symmetric, Asymmetric and Antisvmmetric relations ‘A relation R on a set A is symmetric, if whenever (a,b)eR then (b.a)eR. Tt then follows that Ris not symmetric if we have a and DEA with (a,b)cR but (b.a)eR A relation R on a set A is asymmettic, if whenever (a,b)=R then (b,a)eR. It then follows that Ris not asymmetric if we have some a and b=A with (a.b)=R and then (b,a) bRa 2 ie 4R3 + 3R4 b) A= Set of positive integers, aRb iff a-b: ) Ris not reflective because I-22. »(1.1) (i) Ris irreffective because (a,a)eR for vac (ii) R is symmetric because for ja-bj=2 implies [b-a1=2 -aRb —> bRa Gv) R is not asymmetric because for /$-3}=2 we have (vi) R is not transitive because 5R3, 3RI but SRI because |S-) Prof S.G. Vaidya (Mo 98200 let A=f1, 2,3, 4) and R=I (1,1), (2,2), G31 () Ris not reflective because (4,4) eR {ii) Ris not iereflective because (1,1) ¢R. (Gi) Ris symmetric because whenever (a,b)ER then (ba) )eR (iv) Ris not asymmetric, (9) Ris antisymmetric (si) Ris transitive 4) A= set of all positive integers, aRb iff GCD (a,b)=1. In this case, we say that a and b ere relatively prime. i) Ris not reflective because 3,3 i) Ris not irreflective because (1,1 (Gi) R is symmetric becatsse for (a,8)=1 implies (b,a)=1 - aRb > bRa (iv) Ris not asymmetric because (a,b)=1 then (b.a)=1 “aRb — bRa (6) Ris not antisymmetric because 43 and 3Ra but 423 si) Ris not transitive because 43, R2 but 4R2 because GCDK itis3 =.G.3)eR. ©) AZ: aRbiff axb+l (i) Ris reflective because a Sarl Vac (i) R is not irreflective because 0:01 for 0A (ii) Ris not symmetric because for 2561 does not implies 652 (iv) Ris not asymmetric because for (5,8)eR and also (4,5 (0) R isnot antisymmetric because 4R3 and 3P-4 but 42 (4) Ris not transitive because (11, 10)¢R and (10,9)¢R but (11,9)¢ R R h) A= Set of integers, aR ifa-h (i) Ris not reflective because |1-I|=4 «.(1,1)}2R (ii) Ris irreflective because (@,a)eR for Vac A (iy Ris symmetric because for implies jo (iv) R is not asymmetric because for [S-1/=4 we have I-S|=4 ie. SRI —> IRS. (6) Risnot antisymmetric because SRI and IRS4 but $=1 (vi) R is not transitive because 9RS, SRI but 9RI because |9-1|=4 38.4 Equivalence Relations A relation R on A is called an equivalence relation if it is reflective, symmetric, and transitive 3.9 Equivalence Classes: If R is an equivalence relation on A, then the set Ria) are called equivalence classes of R Some authors denote R(a) by [a] or @ or a 3.9.1 Quotient set: IFR is an equivalence relation on A, then set of all equivalence classes of R is called as quotient set It is denoted by A/R It can be easily verified that two equivalence classes are either disjoint or idemtical The set of all equivalence classes (quotient set) form partition of A Example 3.14 Let A be a set of al triangles inthe plane and R be the relation on A defined as . Fillowe R fa.b\cA~A\ 4 is conaruent to Show thar R is an equivalence relation Prot. G. Vaidya (4) 9320031384, ‘Solution: Step-1: To show that R is reflective Since £20 tangle is congruent to itself we have (2.2)ER VACA. Hence is reflective Sisp-2: To show tha Ris symmetric Henle 2 is congruent fo tangle b then ange b is congruent to triangle a Va A Hence Ris symmetric ‘Step-3: To show that Ris transitive Using ais congruent o triangle band triangle bis congruent to triangle then triangle ais ‘congruent to triangle c. Va,b,c cA. Hence Ris transitive Ris equivalence relation Btample 318 Detemine whether the following relation are reflexive, symmetric or transitive [May-10} GO For As 0.23) R= (0.0.2.0 2.296.250} (2) “is the father of” ‘Solution: (1) Stepe1: To check if Ris reflective Since (aa)@R Vaca, Ris reflective ‘Siep-2; To check ifR is symmetric Since (2,3)ER but (,2)eR, R is not symmetric ‘Step:3: To check if Ris transitive Since (3,1)eR and (1,2)eR but (3,2)eR, Ris not transitive (2) StepeL To check if Ris reflective Since 2 is nor the father of, (a,)€R for ac A. Henc. Ris not reflective Step-2: To check ifR is symmetric. Hea) SR ie b isnot the father of a. But it does not mean that a is not the father of b. Hence R is not symmetric Step-3: To check if Ris transitive KELCGBER and (,c)eR ie. asthe father of b and b isthe father ofc. But i does not mean that a is the father of i.e. (a,c)eR, R is not transitive Frame 316 Let A=2, the set of integers, and let R be defined by @aRb if and only ifaxb. Is Ran equivalence relation? alton: Since asa, Ris reflective. If asb, it ned not follow that bsa 20 Ris not symmetric. Reldeatally, Ris transitive, since ase and bse imply that ase We coe tes ee not an equivalence relation ‘Example 3.17: Let S= Set of integers. Define relation R on ASS as Rb if and only ifaxb (mod 2) Read as “a is congruent to b mod 2." Show that Risen equivalence relation Determine AR. [Apr-Ol, Mey-d7 Dec-09] Solution’ Firs, clear aza (mod 2), Thus Ris reective ‘Second, if a=b (mod 2), then a and ® yield the same remainder when divided by 2, so bea (mod 2) Ris symmetric Tass sUPPOs tht anb (mod 2) and bc (mod 2) Then a,b and eyed the same emuinder stn divided by 2 Thus, xc (mod 2), Hence congruent mod? isan eqshaknne one Equivalences clases are Prof S.G_ Vaidya (M) 9820031384 [ile 881, a, 34 The quotient set A/R = {{0},[1]} z Example 3.18; Let S= Set of integers. Define relation R on A=SxS as ‘ aRb if and only if a=b (mod 5) Read as “a is congruent to b mod 5” ‘Show that R is an equivalence relation. Determine A/R_ {May-L1) Solution: First, clearly a=a (mod 5). Thus R is reflective Second, if a=b (mod 5), then a and b yield the same remainder when divided by 5, so bea (mod 5). Ris symmetric Finally, suppose that when divided by 5. Thus, Equivalences classes are (mod 5) and b=c (mod 5). Then a, b, and ¢ yield the same remainder (mod 5), Hence congnuent mod 5 is an equivalence relation 10,-5, 0, 5, 10 u 4, let 1 [ V8 i 49, ‘The quotient set A/R = {{0}, (1), (21, (31(4}} Example 3,19 Let A=Z and let ncZ° Let R={(a,b)eAxAl azb (mod n)} Show that congruence mod n isan equivalence relation. Solution: First, clearly a=a (mod n). Thus R is reflective a Second, if ab (mod n), then @ and b yield the same remainder when divided by n, so bea (mod 1) Ris symmetric Finally, suppose that ab (mod n) and bsc (mod n). Then a, b, and c yield the same remainder when divided by n. Thus, asc (mod n), Hence congruent mod n is an equivalence relation Example 3.20. Let A=(1.2,3.4} and le R=((1.1), (1.2), (2.1, 2.2) G.4), (43), 83), (4.4); Verify thar R isan equivalence relation Determine VR (Dee-08} Solution: Step: 16 si sas ic oflective Since (2,a)=R ac, R reflective Step-2:To show that Ris symmetric Pr n the given pairs we observe that whenever (a,b)eR then (b.a)=R “a, Hence R symmetric Step-3. To show that R is transitive From the given pairs we observe that whenever (a,B)ER and (b,c)ER then we have (a.e)¢R Fae =A. “Hence R transitive , R is equivalence relation We have Ri 12} = RQ) b Prof S.G_Vaidva (M) 9820031388 Also R(3)=(3,41 = RG) Henee AR={{1.2}. {3.43} Example 3.21: Let S={1, 2, 3, 4} and let A=S«S. Define the following relation R on A! (@b)RG’ b’) ifand only ifa+b=a'+b" So .03, May-05, Nov-06,May-09] {i Show that R is an equivalence relation ii) Compute AMR. Solution’ Step-1 To show that R is reflective Since atb=a*b, we have (a,b)R(a.b). Hence R is reflective, Step-2' To show that R is symmetric sb Le (@b)RC@.b’) implies (a',b")R(ab) Hence R is symmetric bee"-b" fe @.bRIa'N) and (a h'YRUO bY implies (a,b)R( ab") Hence R is transitive Hence R is equivalence relation Now RCI) ) 1.2), 1} R((1,3))"4(1,3), (2,2), Gl) 14), (23), 6.2). 40} R(2,4))=((2.4), 3.3), 29} RGAYREGA), (4,39) ROSACEA} ARC 1D} (0.2, GY}, (0.3), 22), B.D) (4), 2.3), 8.2), 4D} (2,4), (3.3), (4.29), 18,4), (4.39), (4.4) ) Example 3.23: Let S=(1,2,3.4) and ASS. Define the following relation Ron A. (ab)R(a')ifand only if ab'=a"b ‘Show thet Ris an equivalence relation. Compute A/R Solution Siep-1 (2,b)R¢3') ifand only ifab'=a'b Whichis same as (2,b)R(a'\b’) iPand only if a/b=a's , May-08], To show that Ris reflective Since a/b=a>, we have (a.b)R(@,b) Hence Ris eflective Step-2: To show that R is symmetric If abbra’p then a’ ie (a.b)R(a’.b’) implies (,b")R(a,b). Hence R is symmetric, ‘Step-3. To show that Ris transitive If afb=a'/b’ and a’/b'~a"fb” then ard=a"Vb” Le. (2,0)R(a'\b') and (a'.b')R(a",b") implies [AOR 0 J Hence Kis transitive Prof §.G. Vaidya (Mt) 9820031388 Hence Ris equivalence relation Now RE(1 (11), 22), 8.3), (4.493 R((1,2) (1.2), 24)} R«1,3)=(0.3)) R(1,4))=((,4)) RQ, 4,23 RE23))(2.3)} REG.) 1} RG.2)(8,2)} RE3.4))(B.4)} Ras HIY Rud yt4sy, ARE (0), 22), 9), 44)}, 10.2), 24}. (03). (041 (20.4, 12.3)}. 1G.) (G2. 1.4}. 14.3, 1439} 3.10 Properties of the Matrices of a Relation ; We now relate symmetric, asymmetric and Antigymmetric properties ofa relation to properties ofits matrix. Mr=[mj] “The matrix Ma=(m,] of a symmetric relation satisfies the property that ifm, =I then my =L moreover if my =0 then m, =O. My=Mg". Mp is symmetric matrix. «+ The matrix My=[m,] of an asymmetric relation satisfies the property that if m, =1 then m, IER is asymmetric, it follows that mj =0 i, that is the main diggonal of the matrix Ma consists of entries of O's + The matrix My{my) of an Antisymmetric relation satisfies the property that if 4+ (My):o = My, then R is transitive The converse is not true then my Example 3.24 ‘Consider the following matrices, lot ae, poor aor of Lia aj lo o of fo rio foo 11] froon fori a pli 22} gf2oro] sjorrat pfoore roti %ooo1] oor) ooo) Joora} [oeor} [ooo loood rations on Relations Beond © he volatinne from cot & toa set B Then, if we remember that and S are simply subsets of AB, we cain use set operations on R and S ss Db Prof. $6. Vaidya (49 os20031384 * The complement of R, Ris referred to as the Somplementary relation It is of course, a ‘elation from A to B that canbe expressed simply in oroe eras (a.b)e R b if and only if (a,b) eR 1Ne can also form the intersection RS and the union RUS of ihe relations R and §. In relational terms, we sce that (2,6)eRS means that (4,b)ER and (a.b)eS. All our set theoretic ‘operations can be used in this way to produce new relations Example 325 Let A=(1,2,34) and B={a,h0) FeeR((1a) (1). (2b). (20. (3.8), (4.0) and $101.5). (2.6), (4b)} Compute (a) R, b) RAS, (6) RLS, (4) R Solution: We first find AB=((9). (8)... 2.9, 281,22), 3.89, (3,0), 0,0), (4a), (4 b), (4:09) (@) R= 101.0), 22), 0.2),3.0), (48), (4)) OAS = (1,6), 6,b)) (6) RUS = ((1.a) (1b), (2b), 2.0), (31), a), (4.69) R'=(@D, G0, 02), 6.2), 2), .4)} Example 3.26: Let A={a}b,c<.e) and R and § be two relations on A whose corresponding digraphs are shown below. Find (a) RB, () RAS, (6) R! i @R (6.0) () Ros = {(a.by, (), (Qe) (VR 1b), (0), (60). (ed, (Ad) dh Comy Fgh vo (G9), (8), (40), (02), ©.) (6a), (66), (a), (40), (da), (4), (eb), (ea), (ed), eee Se ua 5. Vaya, (0) 920081584 that matsices of Example 322 Let AM(123} an tet Rand S be the relations O° A, Suppose Rand Sare 2 (0. fh a Ma=|° {p 0 0} “Then we ean verify tt fo 1, | Meo|! o 0} Mars Me ample 328 Le Rand Sberelaion defined by the matrices say-031 fo yt 9 foots) P Pe Joo1 0 4 * \u ol 1 . 4 fort 1] 0} Joo 1 1 il fo 1 1 9] lhao o| jy Poset ea order if Ris reflective, Se aay ordered set oF sims © 12 partially Ordered Sets (or 5174 SD Ta R ona set A called © A rte gether with parlor antisymmetric and transitive poset and will Ris a a cae pariah onde. oe may fer 101 PORT we kate an (AR) a. ee Prof S.G. Vaidya © 9820031384 Example 3.29: Let A be a collection of subsets of a set $. The relation < of set inclusion is a partial order on A, so (A.C) isa poset Solution: 1) Ris reflective Since Ais Ay Vaca ARA, —«.Ris reflective 2) Ris antisymmetric TALS Az and Asc Avthen Ar= Az «Ris antisymmetcic 3) Ris transitive Ristransitnc Theorem 3.1: If (A. <) and (B,<) are posets, then (A~B.2) 1$ a poset with pani order = denoted by (a,b) <(2’,b') if asa’ in A and bsb’ in B. Proof. 1. If (a,b)©A-B then (a,b)s(a.b) since ifacain A and beb in B So, , a9 .n) for any n, is called a recurrence relation. A recurrence relation is also called a difference equation ‘To initiate the computation, one must know one (or several) number(s) in the sequence, called the boundary condition(s) or initial condition(s). Mlustration 1: Consider the geometric series (1,5,5%,5...,5"....). Its recurrence relation is n- ith the initial condition Mlustration 2: Consider sequence of numbers known as the Fibonacci numbers. 1,1.23,5.8,13.21.34, Its recurrence relation is ay=an.:+ae2 with the intial conditions ao=1 ay Mlustration 3 (The Tower of Hanoi): It is alittle puzzle invented by French mathematician Edouard Lucas in 1883. There is legend on ‘There are 64 Bold discs of the tapering size, A group of priests is moving them fom one diamond needle is problem, “Tower of Bra! to another diamond needle. There is a third diamond needle for temporary storage. When the move is complete, the tower will crumble and wortd will end, ‘The Tower of Hanoi Problem: “Consider n circular rings of tapering size. These rings are slipped onto a peg withthe largest ring at the bottom. These rings are to be transferred one at 4 time onto another peg, and there is a third peg available on which rings can be left temporarily. I during the course of transferring the rings, no ring may ever be placed on top of a smaller one, in how many moves can these rings be transferred with their celative positions unchanged?” 6-105, Neral Station Complex CW), Fis Floor Prof $.G, Vaidya (9820031388 fample 4.1: If. ‘Solution, Given an.s=2a¢*4, a=3 For n=0 we get a 2agt4, ay=3. Find ay, a2, For n=1 we get ay For n=2 we get a3. Example 4.2. Give an inductive definition of the Factorial sequence ay=n! Solution: n=0, a~0!*1 (o-1)...43.2.19na_1 21 The recurrence relation is a=Mapi, aol nL 42 of Recurrence Relation A function satisfying given recurrence relatos called as solution of the recurrence relation. Some common methods of finding solution of recurrence relationare (2) The Backtracin 2) Total Solutio (3) The Method of Generating Functions Method Homogeneous Solutiont Particular Solution Example 4.3: Determine whether the sequence (,} is e solution of the recurrence relation -2ay.rdq2, for n=2, 3,4, .., where ay=S for every non-negative n. Given difference equation is =2ae4-Bq2, fOr Given solution is o 2 6-105, Nerul Sation Complex (W), First Floor Prof SG. Vaidya ( 9820031384 From (1) and (2) LHS=RHS Hence ay=S is.a solution of given difference equation Example 4.4: Determine whether the sequence (a9) is a solution of the recurrence relation "2291-892, for n=2, 3, 4, ..., where ay=2n for every non-negative n [Apr-04] Solution: Given difference equation is Ae Dagi-tya, for Given solution is ei=2(0-1) and a42°2(n-2) LHS= a,e2n a) RHS= a¢-2ae1-25. “222(01)-2(n-2) n-2-2nt2: From (1) and (2) LHS=RHS Hence (2 nis a solution of given difference equation. Example 4.5: Determine whether the sequence {a} is solution of the recurrence relation 2aet-Bq2, forn=2,3,4,...., where a=2" for every non-negative n_ [Nov-05, May-09] Solution: Given difference equation is for n= 25-2 -Be Given solution is, a) From (1) and (2) LHS#RHS Hence a¢-2" is nota solution of given diference equation Example 4.6; Is the sequence {a,} a solution of the recurrence relation 6-105, Nena Station Complex (W), Fits Floot Prof $.G. Vaidya (M) 9820031384 a= Bagt-16aq2, faa"? [Nov-04, May-08] Solution: Given difference equation is 8=Baq3-16852 Given solution is 7 na eam (0-14) and ay2=(0-2)4° nat a RHS* 8341-16a¢2 “8»(0-1)P4"-16x(0-2)*4 ={8(n-1)'4-16x(n-2)' x4" HG ar Q) From (1) and (2) LHS¢RHS Hence a= n'4” is not a solution of given difference equation Example 4.7; The solution of the recurrence relation Coan+Ciay.1+Cxte2=fn) is 3°+4°+2, Given that f(n)=6 for all n. Determine Co, Ci, C2 [May-05, Dec-08, May-10) Solution: Given recurrence relation is Cote+C a0 2-6 (given fln)=6) Also given solution is ag=3"+4"42 Cof3™442)HC AST HAH 2)HCHI™ 4242/6 Consider the coefficients of 3°, 4" and constants from both the sides. ¢ (Cor Ss & 38 4 Cor G+ Syr2(CotCirCah6 Comparing the coefficients of 3", 4° and constants from both the sides we get, Example 4.8: The solution of the recurrence relation Coay+C ay¥ Cady2™f{n) is 24345, Given that f{n)=40 for all n, Determine Cp, C1, C2 [May-06, May-11} Solution: Given recurrence relation is Coag+Ciae-1+Catq2=40 (given f{n)=40) Also given solution is a,=2"+3"*5 Cof234 SHC AMI S)+C ("I") AO Consider the coefficients of 2", 3"and constants from both the sides, 6-105, Neral Staion Complex (W, First Floor Prof S.G. Vaidya (M0 9820031384 Sysexreieyy40 ‘Comparing the coefficients of 3", 4" and constants from both the sides we get, Cot ¢ Sino, cot Ses Sino, and 5(CotC+Ca}40 24 a9 Solving these equations, we get Co=4, C; 0, C24 4.2.1: The Backtracing Mi In backtracing method we substitute the value of a, in terns of previous ax's. Repeat the procedure till we get intial condition, Simplify to get the solution nce {ae} 83 2 solution of the Example 4.9: Use baci recurrence relation a=ay.i73, 21° Determine the sequence {2,} as a solution of the recurrence relation a Solution: Given Lays) anat3}H#23 act If we take, rn-1, then we write A ayseaytleel) 3 a= ay+3n- 3824306 3530-1 ‘The solution is ay-3n-1 ml Example 4.10; Use back tracing method to find the solution of the recurrence relation bby=2bp+1, b1=7. Solution: Given difference equation is ber2bn*l, b1=7 dye2l2bea4l 41 o2° bgt 24] (6-105, Nena Suion Complex CW), Ftst Floor Prof $.G. Vaidya (M) 9820031384 =2* bast 4241 by =2" nel 4 sample 4.11; Using backtracking method Show that 31 j¢ solution of the recurrence by] for nz] with by relation by jon, Given difference bby=3aba-l =14 Now bi=3bes [beat o3% byl =3' baa 3+) bya"! beGM43™ 4. $341) by ITED, AHIR bo PHIM. 341) Example 4.12:(Tower of Hanoi Problem) State the “Tower of Hanoi” Problem and obtain, the corresponding recurrence relation indicating the initial conditions. Solve the recurrence relation [Nov-04, Nov-05, May-08, May-09, Dec-09] (6-105, Nena Staton Complex (W), Fust Floor ao Prof SG Vaidya 0) 9820031384 Solution: “Consider n circular i the largest ring at the bottom, ‘These 1 of tapering size. These rings are slipp4 onto a peg with are to be transferred one at atime onto another peg, and there is a third peg available on which rings can be left temporarily. I; during the course ¥ of transferring the rings, no sing may ever be placed on top of a smaller one, in how many ‘moves can these rings be transferred with ther rela /e positions unchanged” Let us consider all she ings are on sg A initially. These rings are to be transferred to peg B. Peg C is available fcr temporary storage Let the number of moves required with m rings. *. Let us formulate recurrence relation as follows (® n-1, ie. there is only one ring on peg A. Onl; :nove is required to transfer it to peg B. Ui) n-2, Le there are cu sing un peg A. Move the smaller to peg C, then: move the larger {0 pey B and finally the smaller from peg C to peg B. Number of moves required is3 3, ‘e. there are three ring on peg A. Move the smallest to peg B, move the middle ii) ‘© peg C, then move the smallest from peg B to peg C, then move the largest from Peg A to peg B, move the smallest from peg C to peg A, move the middle from peg C to peg B, and finally the smallest from peg A to peg B. Number of moves required is 7. -. ay=7 (dv) Transfer the upper (n-1) rings ftom peg A to peg C using above rule. Number of ‘moves required for this transfer is ay. Then transfer the largest ring fiom peg A to eg B. Number of moves required for this transfer is 1, and then transfer the Ca-1) rings from peg C to peg B using above rule. Number of moves requited for this transfer is a4, Hence total number of moves required is ae 1+ ae1 i Zaps D ‘The required recurrence relation is. ay"2ay.*1 with a.=1 Solution by backtracing method 6-105, Nerul Station Complex (W), Fst Floor L i Prof S.G. Vaidya (M4) 9820031384 anm2 14224274... 241 aga2-l nel Eu es Obtain the recurrence relation and initial conditions to find the maximum number of regions defined by n lines in a plane. Assume thatthe Fines are not parallel and lines not intersecting at one point when n>2, Solve the recurrence relation {[May-03 Rev, Nov-04, Nov-06, Dec-08, May-10, Dec-10, May-11] ‘Solusign: This problem was first solved in 1826, by Swiss Mathematician Jacob Steinner Let the maximum number regions of the plane when there are n lines in the plane be Ls Let us formulate recurrence relation as follo (i) 10, ie. there is no line in the plane. ay wo ie. there is one line in the plane. This line divides the plane into two regions, (iit) n-2, ie. there are two lines inthe plane, For maximum number of regions, these lines must be intersecting to each other. These lines divide the plane into four regions a= Gv) n3, ie, there are three lines in the plane. For maximum number of regions, no nwo Jines will be parallel or concurrent. These lines divide the plane into seven regions a7, (v) nen, ie. there are n lines in the plane. There were (0-1) lines in the plane and the ‘maximum number regions ofthe plane were ay, Draw'the nth Fine. It interests the earlier lines and number of regions increases by n a= aan, “The required recurrence relation is. ay= ern with ao=1 Solution by backtracing method: a= aert y= aet(n-l)}H(n) y= das n-2}4( 1 }H(n) st 6-105, Nerul Station Complex (W), First Floor me: Prof S.G. Vaidya (4 9820031384 a= agt 424 + (n-2)H(ne1) (0) a= 14S, where Si=142+ + (n-2)4(nel)H(n) nin) lethod of Homo; A Linear Recurrence Relations with Constant Coefficients is a recurrence relation of the form CoastCians . 4Ceay Where all the Ci’s are constants in) a For example, 3ay-Sau+2a,.=n°+$ ia linear recurrence relation with constant coefficients ren 9. then abave equation (1) becomes Cott Cider $Came=0 Q This equation is called as homogeneous equation. The solution of homogeneous equation is called as homogeneous solution, Depending upon the form of fn) we will get the particular solution, The total solution of s inear recurrence relation is equals to the sum of homogeneous solution and particular solution 4.2.2.1 Homogeneous solution To find the homogeneous solution we solve the characteristic equation, which is the polynomial equation Coley Cr (a). tad oa © Solutions of characteristic equation (3) are called as characteristic roots. Depending upon the nature of characteristics roots we have the comesponding form of the homogeneous solution. Our discussion is limited to real roots only Sr] Nature Characteristic Roots Homogeneous Solution No, T Real and distinet = Amy BG Say my and my 2 ‘Real and equal a= (AmB Say m=mz=m 3 Two roots real and equal and one distinct = (AmB) Cm) ‘Say my=m=m and my perth 6-105, Neral Station Complex (W), Firs loo: Prof $.G. Vaidya (9820031588 with initial 1 ig the solution of the recurrence relation aya, Solution: Given difference equation is The n (2), we get arAsB 2AsB Take n=1 in (2), we get ay AC)'+B02}! @ Solving (3) and (4) we get, Be The solution is given by aye -1 1) #32)" i \ (-1) 743 (2)" ) — Bi Exgifiple 4.15: What is the solution vu. recurrence relation ay=3aq1-2a52, with initial !Nov-05, Nov-06, May-0 Aonsitiona Solution: Given difference equation is 243i 2a te Sao +28 The characteristic equation is (onthe ml The general solution is 6-108. Nerul Staion Co plex (W).Ftst Fleer Prof S.G. Vaidya) 9820091384 a= A(an)) BC: aA (1) +B)" To find Aand B use the initial conditions aj=S and ay=3 Take n=1 in (2), we get y' +BQ)! SA 428, Take n=2 in (2), we get ACI) BQ) 3=A 4B Solving (3) and (4) we get, A=7, B= -1 ‘The solution is given by aye7(1) "1 (2)" a=7-(2)" ni Example 4.16: Solve the recurrence relation for the Fibonacci sequence [Nov-06, May-08] tea with a= 1 and oR Give the recurrence relation for the following sequence {a} indicating the initial conditions. Solve the recurrence relation 1,1,2,3,5,8, [May-05] ‘Solution: Given recurrence relation is &j=a.1+452 with a=1 and es ‘The corresponding characteristic equation is oor —bd hee Ce which has two distinct roots ™, S-t] 0 ‘The two constants Ay and Az can be determined from the boundary conditions =I and. by solving the two equations “Arras 6-105, Nerul Station Comples (Fist Floot Prof SG. Vaidja (Mf) 9820031384 Example 4.17 Find the solution to the recurrence relation aj-Gaq1-11ag2* 6ay.3 and ar=15 [Nov-04, De with the initial conditions a Solution’ Given recurrence relation is a"Gatyi-11 3° Bay The characteristic equation is m’-om* (1 \(o-2}(m-3} m2 and m=3 ~ i.e. roots of the characteristic equation are real and equal. The solution is given by a,A(mj)"*B(m,)"*C(ms)" a AC BQY+CGY (2) Use initial conditions to find the arbitrary constants A, B, C Substitute n=0 in equation (2), we get ay-A*B*C — «.2“A+B+C in equation (2), we a=A+2B43C = S=A+2B+3C +4B+9C.. IS=AMB+9C in equation (2), we a; 1, and Solving these equations we get A=1 a)" 20 Example 4.18: Find the solution of the recurrence relation defined by ot-2aq2 with ar=5 and a: [Dee-02 1d] Solution: Given recurrence relation is ae 3ap.i+2ay2=0 (1) It is a difference equation of Hence the characteristic equation is given by 6-105, Nerul Staion Complex (W), Fits Floor 4 0 Prof. G. Vaidya 0 9820031364. mP-3m+2=0 —».(m-2Xm-1)-0 m=2, and m= Roots of the characteristic equation are real and distinct, Hence solution of the homogeneous equation is given by a=A(em)"*B(m)" ar AQ)B()" a= AQ)4B —(2) Where A, B are arbitrary constants. Use intial conditions to find A, B. @) @) ay=5_ Take n= in equation (2), we get Take n=2 in equation (2). we get Solve (3) and (4) we get A= -1 and B=7. Hence the required solution is age -(2)"=7 21 1-983. with inital Example 4.19 What is the solution of the recurrence ce! condition ac=1, a/=6 [May.06, May-11) Solution. Given recurrence relation is a:=62i-9en2 y-625.°9a, The characteristic equation is m'-6m#9=0..(mm-3)2=0-.m=3 and ‘le roots ofthe characteristic equation are real and equal The solution is given by aar(An+B)m" ‘AntB\(3)" Use inital conditions to find A and B ‘Take n=0 in equation (2), we get aye(A04B)(3)?. Bet 46, Taken=1 in equation (2), we get a:=(Ax1+B)(3)' A*B=2 2 1 The solution is age(1+n)(3)" a Example 4.20 © What is the solution of the recurrence relation ay~2an4-dqa, with initial ‘condition ay=1 5, ar=3, [Dec-07, Dec-09} Solution’ Given recurrence relation is 24°22 a9-2aertdy a0 a The characteristic equation is m*-2m-+I~0 (m:-1)?-0 m=] and m=1 i.e roots of the characteristic equation are real and equal ‘The solution is given by a,=(An+B)m" ay=(An+B\(1)" Use initial conditions to find Aand B (G:108. Nerul Stauon Comples (W). Fist Floor Prof $ G Vaidya (M4) 9820031585 in equation (2). We get ay~(Ax1+B)(1)! @) 5, take n=3 in equation (2), We get a=(A»2+BY 1)? ANB = (4) Solving (3) and (4) we wet A=1.5, B-0 ‘The solution is ay=(1.Sn+0\(1)" age Sa ne] Example 4.21. Solve the recurrence relation, Cy+6Cy+12Cu2*8C 430 with the boundary conditions Cy=1, Ci= 2, and C=8 [Dee-00} Soh Given recurrence relation is, Ce#6Cy1*12Cy#8C 30, 23 y It isa difference equation of order 3. Hence the characteristic equation is given by m's6m+12m+ (m=2)'=0 :.m=-2 isa triple root Roors ot “he characteristic equation are real and equal Hence solution of the hemoweneous Coe(An?+BneC im)" Cu(An?+Bn+C\(-2)" —() Where 4. Band C are arbitrary constants. Use initial conditions to find A. B. C ce c C8 Take ‘Take n=0 in equation (2), we get 2 Take w in equation (2). we get 2 in equation (2), we get Solve) +4) and (5) we get ls dati Hence the required solution is bg. 1Adea, di Ly [May-00] Example 4.22: Solve the recurrence relation, 6 Solution: Given recurrence relation is Gandy -4dns dd 444, » It is a difference equation of order 2. Hence the characteristic equation is given by Ama=0 -(m.2)7=0 m= 2is.a double root Roots of the characteristic equation are real and equal. Hence solution of the homogeneous equation is given by de=(An*B)(m)" (An By(2)" = G-105, Neral Staion Comples (W), Firs Floor wv Where A and B are arbitrary constants. Use init iT Take n=1 in equation (2), we get 44 eT Taken=2 in equation (2) we get Solve @) and (8) we get A= and B= Hence the required solution is 2.2.2 Particular Soluti Prof SG. Valtys 9820031384 ial conditions to find A, B @ ml To find the particular solution, observe the right hand side ie ffn) Depending upon CConstant= Exponential expression } ed? 42x4% ete [Sr |For of RHS Form of Particular Solution |_No. se ee eee ee t J constant jana | | Le: -3, 42, 11/2 ete | General constant | ES 1 == ane | Gena nea expression ‘express ay = An+Bn+C = PS, noms etc | General quadratic expression | poe | all | General constant - Same exponential | expression Le Qnt5) -4", ansatete i | (The base of exp. Expression is not I a characteristic root) | (The base of exp. Expression is not | expression I 2 characteristic root) 5 | Linear expression = Exponential | =(An=Byxa™ 1 General linear expression » Same exponential expression © | Quadratic expression expression ie Qn?+5) x4" an?x4% ere Exponential | (The base of exp. Expression is not aan Bac) General quadrati~ expression ~ Same | exponential exression j | a characteristic oot) oe ae Example 4.23: Find the particular solution of @-54.1+622 Solution: Given recurrence relation is a:-Sar1*6: a 6-105, Nerul Suton Complex (W), Fest Fleor Prof S.G. Vaidya (M) 9820031384 ‘or the particular solution, observe the «-hs of the equation (1) tis constant. «Consider the particular solution = general constant ae aA, and a2=A, Substitute these values in equation (1), we get A-SA+6) 2 pasarche\ 1 1 r Therefore the paricular solutions (=! gat 2 that 2 2 elarae ED ExSimple 4.24: Find the particular solution of ay-3a.i+3aea¢a,34 [May-03, May.08 Solution: Given recurrence relation is a-3a,..+3a,.2+a.s=4 4) For the particular solution, observe the rh ofthe equation (1), peeti Ie ig = 4= constam. Consider the pantculr solution = eneral constant \ wR . TER AIMA tae A and 3° anf Substitute these values in equation (1), we get ORR, peveesiren) Oo%ha ‘ pehek \ 7/IK* ote ‘Therefore he partohlar Cithor is a2 4.25: Find the panicular solution of a¢+5a,1+6a,2=37 [May-02, May-08, Dec-08] ‘Solution: Given recurrence relation is ar*Sa.1+62,2=37, a) For the particular solution, observe the r hs of the equation (1), tis = 37° a quadratic expression ‘Consider the particular solution = general quadratic expression, APSBHC BMAP +Br+C Jai A(r-1) +B (6-1) +Cand(a=A (r-2)°+B(e-2)+C Substitute these values Tm equation {I}, we get [AP sBrC}S[A-)*+B(-1)CH6(A-2)4B(r-2)C) 12Ar°4(-34A+12B)r*+29A-17B+12C)=37 ar Comparing the coefficients ofr, rand constants from both sides, we get, 12A63, -34A+12B=0 and 29A-17B+124 6-105, Nerl Staion Complex (W), Fast Foor a o Prof SG Vaidya @ 9820031384 Lgl? ous Solving these equations we yet az! pal? ca!15 8 a 2 4 24 288 ‘Therefore, the particular solution is a= 12417, U5 4) "24" 288 Example 4.26; Find the particular solution ofy*Saq1462n2~42x4" ——_ (Nov-05, May-11] Solution: Given recurrence relation is a+54y. +6: For the particular solution, observe the rhs of the «~ation (1) tis = 424" 2a... (1) (aconstant)x 4° Consider the panicular solution neral constant)-4" Manca BHAA, ByaeAcd™, and ays And? Substitute these values in equation (1), we get [Aas}5[Aca™Ys6 Axa" 42.4? 5 Ana” 6 Ande S 5g 42a? a6 5.6 145.6. a6 A Therefore, the particular solution is. a! Wn get Example 427; Find the paricular solution of asta3r<2" _[Nov-04, Apr-04, Dee-09] Solution: Given recurrence relation is atacy=3r-2 (1) For the particular solution, observe the rhs ofthe equation (1) Itis = 3rx2" = (a linear expression)x 2 Consider the particular solution = (a general linear expression) 2" 2 =(ArB)x2" 8=(APB).2" and a1=[A(l-1)-B)s2"" Substitute these values in equation (1), we get (APB) [ACER] a3 p.2" (APB )2"[A(e-1) Be 2 a3p2! (6-108, Nerul Staion Comps (W) Firs Fae Prof SG Vaidya (M) 98200313 (ArsB)TA(e1)+B)> : eel 2 an pt ales 2 5 2‘ Comparing the coefficients of rand constants from both sides, we yet 3a and 3p. z Solving these equations we get ‘Therefore, the particular solution is a,” Solution Given recurrence relation is a+a,.=(47+2r+6)«3" ay For the particular solution, observe the rhs of the equation (1) It is = (4+2r+6)-3' = (a quadratic expression).3" ‘Consider the particular solution = (a general quadratic expression) -3 a(P'(AP*Br+C)<3 anu ae) =[A(r)#B1)+C}3"" ae(As+Br+C) 3 Substitute these values in equation (1), we get (APB) LAL IP BEe 1) +O) 3 (4426) (APB C)TACeH BIE I)HC)+“(4P2r6) Comparing the coefficients of rand constants from both sides, we get A=3, Be3 and C= anion ay “Therefore, the pancular solution is a/"% Geran js UF fi is ofthe form (Ba = By! + B,yn« Bj” where m isthe characteristic root of ‘multiplicity k then the particulay solution is ofthe form (d.n° «Ayn =- 4, ah 16-105. Nerul Stauon Complex (W, Firs Floor As « Prol:S.G Vaidya (9820031384 Example 4.29: Find the particular solution of ay 2a (322 Solution’ Given recurrence relation is ay-2aq1=3x2" io) Forthe arcularsluion, observe the rhs ofthe equation (I) Note tat 2a charatrie "00% (Of multiplicity 1), Hence the general form ofthe panicular solution is Ame?" a= Aaxzt a= Ana, a A(ne1) Substitute these values in equation (1), we get Amx2*. A(ne])x2!= 328 Solving this we yet as3 Therefore, the panicular solution is 4!"'= 3n-2" Example 4.30: Find the particular solution of y-6ay1+98,"(n+)<3" (Dee-07] Solution: Given recurrence relations 6a s#9aq3=(ne1)-¥ “ For the paiculr solution, abserve the rh ofthe equation (1) Noe that 3 i characterise "00! (of mukiplicity 2) Hence the general form ofthe particular selation i 2, = n}(An+B)3" 2 WADE)", a= (0-1) (ACOI)SB YS", ay Substitute these values in equation (1), we get (AN*B)3™6x(- 1 [AC] HB)-39x( 2)"[A(n-2)+B)<3"* 2)A(n-2)+B 3"? = (m1), Solving this we get A~ ‘Therefore, the particular solution is a40”= nix {4 8 sn Example 431: Find the panicular solution of ay4aq+daqa=(n1).2 Solution: Given recurrence relation is a-q144q3=(n+ 12" ay For the particular soition, observe the rhs oF the equation (1) Note that’ sa characteristic ‘00% (of muliplicty 2) Hence the general form of the particular solution is 4.0% n?(AntB)-o" 6-105, Neral Savon Complex (W), First Flor Prof SG Vaidya (M) 982003 384 =O AMBY2" ayy (1) [ANB aga (n-2)'[Aln-2)}4B)2"? Substitute these values in equation (1), w n(An+B)+2"-44(n-1) [ln +B) <2" +4 x(n (Ac-2)¢B 2" m4) Example 4.32: Find the particular solution of Solution Given recurrence relstionis a ve the rh s of the equation (1) Note that ‘oot (of multiplicity 2). Hence the general form of the particular solution is a= Ant age An, agi A(nel) Ane Substitute these values in equation (1), we get +AlneI)F+24A(0-2)°= 7 An Solving this we get A: ‘Therefore, the particular solution is ay"= Example 4.33: Find the particular solution of 25-5 2y 16a = 2 Solution. Given recurrence relation is a-Say.1 "8a a For the particular solution, observe the rh.s of the equation (1). Note that 2 is a character root (of mul iplicity 1). Hence the general form of the particular solution is Anes RatC 'B(n-IHC, ay in equation (1), we get S[A(aet 2" +B 1)°C}+6[A(n-2)-2" 6-108, Neel Staion Comples (W). First Floor 1 Prof SG Vaidya (4 9820031384 Solving this we get A= -2, B 4 ‘Therefore, the particular solution is a= -n: Example 4.34: Find the solution of the recurrence relation defined by bum 4ba.r+1 with bi=4 as initial condition [Noy-03 Old) Solution: The given recurrence relation is by: ab Step(i), To find the solution ofhomogeneous equation ‘The characteristic equation is ms The solution is ay = A¢4y Step (i) : To determine the particular solution Observe the rhs Itis=1= constant. Substituting this into (1), we have BB: which gives D=S" and thus, 3 Step (i): The total solution of the difference equation is simply’ the sum of the homogeneous and particular solutions, Thus 1 aay Hays From boundary condition, the constant A is determin (3 pte (a, ay J Example 4.35 Use the method of homogeneous solutions and particular solution ta salve 3aq=Saq1-2an.2¢n with initial condition ([Dec-98, May-10] Solution: Given recurrence relation is 3ay=Saq-2aq2tn 3a, Step-1) To find solution of the homogeneous equation Cn (1) For homogeneous equation take ths=0 in equation (1), we get 105, Nerul Staton Comples (W), Fits Foor Prof SG Vaidya (M) 9820031384 3a Stes +289250 Iisa difference equation of order 2, Hence the characteristic equation is given by 3m?-Sm+2=0 .(m-1\3m-2} mand m=? 3 ‘The characteristic roots are real and distinet. Hence solution of the homogeneous equation is siven by aa¢m)" +B(my" ‘>= atyy +3{2] Where A, B are arbitrary constants, Step-2) To find the particular solution Note that 1 is a characteristic root (of multiplicity 1). Hence the general form of the particular solution isa" n(Cn+D) a= n[CnD],—— ayin(n-){Cin-1}+D], Substitute these values in equation (1), we 3afCn+D]-5(n-1)[Cla-1)+D]+2(n-2)[Cln-2)+D-n (n-2)[C(n-2}¢D] Which on simplification gives 2Cn+3C+D ea Comparing the coefficient of like terms on both sides we get 2C=1 and 3+ Solving these equations we get C: 2) Step-3) «The total solution is given by aoa on 7G a the initial conditions, in equation (4), we get AYB = 6-105, Nerul Saxon Complex (W), First Floor Prof SG Vaidya (9820031384 Take 6) in equation (4), weget —A¥ 2 B= Solve (5) and (6) we get A= 8 and 8-9 2) oofn-2) Hence the require solution is ant 3 (io-3), m0 Example 4.36 Find the solution of the recurrence relation ‘44°200aq°100 with initial condition ao=1 [May-03 Old, May-06) Solution: Given recurrence relation is 008y.1-100 4-200a,1= -100 Step-1) To find solution of the homogeneous equation. For homogeneous equation take rhs=0 in equation (1), we get 24-2008 = 0 wisa difference equation of order |. Hence the characteristic equation is given by m-200=0 .m=200 Hence solution of the homogeneous equation is given by (2) Where A is an arbitrary constant Step-2) To find the particular solution For pantcular solution, observe the chs of (1), Hence take a Hence a=B, an1B, Substitute these values in equation (1), we get B-200B--i00 .B=100/199 100/199 stant. '=B = a general constant 0 Step-3) ©. The total solution is given by ae a 400) 100 a= A(200)"+. “ 95 i To find A, use initial condition ‘a=1. Take n=0 in equation (4) we get 6-105, Nerul Suton Complex (W), Fis Floor Prof S. G Vaidya (M) 982003 199 199 ie tremens 2 agoyi0 1 ®onoy 12 eS) Mi 199 y 199 Example 4.37; Solve the difference equation 624, = m3 with the boundary condition ay*3 (May-05] Solution: Given difference equation is 1) Step i) To find the solution of the homogeneous equation antag The characteristic equation is m0 The solution is ay AC-2)" Step (ii) To find the particular solution Observe the rhs. Itis n*3= linear expression. a “general linear expression, ay = n+ Substituting this into the difference equation, we have — Bn+C+2[B(n-1)+C]=n: which gives 3Bn+3C-28 = n+3 ‘Comparing the coefficients of n and the constant terms on the two sides of this equation, we l have 3B=) and 3C-2B=3 thatis —B. 9 nou and thu en. a*9 Step (iy The total solution ofthe difference equation is simply the sum of the homogeneous a2" -(2) Example 4.38: Solve the following recurrence relation 26 Tae #84,2°16F*=10Fr40 [May-03 Oi} 6-105, Nenu Suation Complex (W). Fist Foor Prof § G. Vaidya (9920031384 at Tay.y8ap2=16r*-10r+40 Step) To find oition ofthe homogenous equation For homogeneous equation ke shs-0 in equation (), we get + Ta+8ae2= 0 eis derene equation o order2. Hence the charac eution i given by Pena ir ms7ms8=0 a The characteris roots ae real an dstne. lence soution of he homogeneous eqation is siven by a= A(m)' *B(msy aed cte8T , zy Where A. B are arbitrary constants, ‘Step(2): To find the particular solution For particular solution, observe the rhs of (1) Tis = 161'-10r+40 = quadratic expression, Hence take 2,9" Cr?Dr+E= a general quadratic expression Cle-1)+D¢r-1)46 and aa C(r-2)"+D(1-2)6E, Substitute these values in equation (1), we get (CP+Dr+E}+T1C-1)*+D(- IHE}+8{C(e-2)*4D(-2}4E}= 16F-10r+40 16CP*(16D-46C)r+(16E+39C-23D)= 161?-10r+40 Henee a= CP+DrE , a= ‘Comparing the coefficients of like terms on both sides, we get 1eC=16, 16D-46C=-10, 9,2 oS el a a ae e ‘Step (3) .. The total solution is given by am ala) 101 (6-105, Nena Stavon Complex (W), Fist Flor Prof $.G. Vaidya (M) 9820031388 Example 4,39: Solve the following difference equation a¢¢Saj*6a.23r°, [May-02) Solution: Given recurrence relation is a/+5a,+6a.2°37. ay ‘StepC1} To find solution of the homogeneous equation For homogeneous equation take rhs=0 in equation (1), we get 8,453,460, Iisa difference equation of order 2, Hence the characteristic equation is given by missm+ The characteristic roots are real and distinct, Hence solution of the homogeneous equation is zoenby Atm)" +B(my (3) 4B(-2) Where A, B are arbitrary constants A 2) Siep(2) To find the particular solution For particular solution, observe the ths of (1) quadratic expression Hence take a,"'= Cr?+Dr+E= a general quadratic expression Hence a= CPD r+E , a= C(F-1)+D(r1)+E and a.2° C(r-2)°+D(r-2)4E. Substitute these values in equation (1), we get [Cr?+Dr-E}+5[C(r-1)*+D(0-1)+E}+6|C(r-2)*+D(r-2)+EJ= 31? 12CF4(-34C+12D)r+(29C-17D+12E)=37° Comparing the coefficients of, rand constants from both sides, we get “34C+12D-0 and 29C-17D+12E=0 Solving these equations we get C= Therefore, the particular solution is a= 1.417, ‘Step(3) «The total solution is given by a= a) +a, an AC +812) 6-105, Nema Station Comples (W, First Floor Prot SG. Vaidya” (M4 9820031384 Example 4.40: Solve the difference equation ay+2ays +ay ‘Solution’ Given difference equation is ct 2aq Hays 2° [May-10] ‘Step (3): To find the solution of homogeneous equation, ‘The characteristic equation is m?42m+1=0 ‘The homogeneous solution is ad = (AnSB)-1)" Step (i): To find the particular solution. Observe the rhs. It is 2" ‘The particular solution is of the Form Cx2" My CX2" ay =Cx2" Substitute these values in equation (1). we vet Pada 4 Step (ii): The total solution [AnSBY-1)" + 2" c Example 4612 Let bo=5 and for n>0, let by=10b,1*9, Show that bye6.10"-] What are the first three terms ofthe sequence {ba} [Nov-03 Rev] Solution: Given difference equation is ‘The characteristic equation is ‘The homogeneous solution is ba= AC1O)" ‘The particular solution is found by trying a solution of the form Bn. Since Ba 10B(a-1)+9 B: ‘The general solution is by a(toyt Toffind A, use the intial condition by=S. We get bo=A(10)"-1 “1 A=6 The solution is by=6.10"1 599 First three terms in the sequence are -by=5, bi=59, 4.5: Finite Differences We will study following finite differences 1) Forward difference, 2) Backward difference 103 6-105, Nerul Station Complex (W), Fra Floce Prof SG. Vaidsa (M983 4.5L: Forward Differences Let an be a sequence {ay} of a real number the forward difference of the sequence defined as follows (@) The first order forward difference is 81 ii) The 2" ovder forw: d difference is (ii) The k* order forward difference is Example 4.42: Find Aa, where (i) 3423 , (i) a, [Apr-o4] Solution (i) Given 3 ay.re3 Angmaopir 823-30 t neta Sau" Oafi-de= n+} +7 (427) (ity Given a= +n 4 Line) a DP Lynne yan Let a3n'+n+2, Find A's, where k equals to (1)2, (2)3, (3) 4 [May-05, May-11) m4 ages ne) A 1H Bia-1)*4(ne 1)42-Gn?ene3) G-10. Nenu! Sation Comples (W) Fits! Floor Prof SG. Vaidya @¥) 9820031384 0 (3) To find Aa, We know a*a,=0 A 2g91"0 0 Example 4.44 Let a= Find A< ipa Le n> ytd dee Solution: éf Consider Ons and ay, (ii) Consider Aap sara; Jah ‘Consider n22-—ay=243 and ag.y=2""!43 Bate ty 232%) on Example 4.45: Let {a} and {b,) be sequence of real numbers then show that Alanby)~ ay-sA(by}* ByACQ,) Solution A(2yb,) = aasabyat- aby = Byibaet Boe Dut eBay ie 6-103, Nerul Station Complex (W), Fin Floor Prof SG Vaidya _(M) 9820031 (Bp-ba) "by -4) Aba} bya) Let an be a sequence {a,} ofa real number the forward difference of the sequence {aq defined as follows (i) The first order backward difference is. Va, (ii) The 2” order forward difference is Va, eV (ii) The k* order forward difference is. Veag=V"!ay-V!aq Example 4.46: Find Vay where (3) 4-5 , (i a Sa [No fil) Given a= asm Sle) (ae 1)42 S(n-1)(n-1)+2] “10n-6 Example 4.47 Let a 3 Find where k equalsto (1)2.(2)3.(3)4 —(Dee-09) Solution Given ay=2n?-3 ant 2(n- 13 (n-1)7-3-(2n2.3) (1) To find Via Va-Van 6-105, Nerul Sauon Complex (W), First Floor o Prof $6. Vaidya (4) 9820031384 =4 2) To find Va, We know Va,=4 Van Vay Vu 44 =o (3) To find V*eq We know Vag=0 Way =0 Vag V ag Wan =0-0 Spam Here |o 8 \ Ton43 mag Find Va (May.06, May-08,Dec-08] ‘Solution: (i) Consider Isns2 ‘=O and a4.1=0 Vae=antes =0-0 -0 (ii) Consider n=3 ay=0 and aj=2°43, Vay=a1-2: 943.0 =25/8 ome Pees Vacnaa-aoa -O49)-2%3) = Thus 6-105, Nero Siation Compl (W) First Floor Prof SG Vaidya (Mt) 9820031388 se 4.49: Let {9} and {be} be sequence of real numbers then Show that Vea.) ays P(bs)* by VA) [May=09} Lution: V{asba) = aaby- Snibas [Nov-04, VITI De Esumple 4.50 Show thar iy : 05) 2V ant Vay = Ay 2(Ag- 1) *V (2p Bt) api Var. 2a-1*(BarBet ef a> Bo) Exercis the a plane is divided by n ovals It is clear that ay=2, recurrence relation is ay=ay.1+2(n-1) (2) Find first five terms in the sequence {a,} whose recurrence relation is +3, with ao=1 Say -3ay2, with ay 29,124,133] (Gil) agar, with ag=1 6-105, Nerul Sato Complex (W), Fist Face o = Prof SG. Vaidya @) 9820031388 (3) Determine whether the sequence {a,} is 2 solution of the recurrence relation ay=28q:1-802 1n>2 where a,=3n for every non ns ive n Answer the seme question when [Dec-02 Old) [Ans: Yes, Yes] (4) Determine whether the sequence {ay} isa solution of the recurrence relation n1*2ana+20-9 n22 where a= - n¥2 for every non negative n [Ans: Yes] (0) is 2°35, Given that [May-06) (5) The solution of the recurrence relation Coan+Cit,+Caa fin)=40 for all n Determine Cy, C1, C2 [Ans G) (6) The solution of the recurrence relation Coae*Ci2y-1*C2aq2=A(n) is 4"+S"*6. Given that 0 for all n Determine C), Cs, Cs finy (7) Using backtracking method Show that iL for nat with, (8) Use backtracking method to solve ayeay.-+n. with ao~1 [Ans (9) Use backtracking method to solve aj=aqi-+4, with 203. [Ans: ae3+4n ] (10) Use backtracking method to solve ayay.:+1, with ay=1 [Ans aet+n ) (10) What is the solution of the recurrence relation 2a, 1-54.20 with initial condition ass. (11 (Rabbits and Fibonacci Numbers) A young pair of rabbits one of exch sex is placed on an island Once they are two months old, eacl. pair produces another pair each month. Find the recurrence relation for the number of pairs of rabbits on island after 8 months. Assume that no rabbit ever dies. Also solve the recurrence relation (12) Find the recurrence relation and give the initial conditions of the number of bit strings of length n that do not have any two consecutive 0's. How many such bit strings are there of lenath five? [May-02] (13) Find the solution to the recurrence relation @y-3ay.:*4aq3 = 0 with the initial conditions aS < I ane: !? yoy of 2) a and ar=S. ans: [ nat) (2p [2 ) ea 1 ae tg 6-105, Nerul Stauon Complex (0). Fist Floor Prof SG. Vaid (0 20081384 (04) Solve the difference equation ay*6ay+125y.248a,.=0 with the boundary conditions, 1 Sat ay-1, a, -2, and as-8 [ans . ¢ (15) Find the total solution of a,-2: particular solution 6 Solve the difference equation (17) Obtain the recurrence relation for number of regions of a plane when there are 1 concentric circles init Also solve the recurrence relation [AnS: a= yi! ay = n+l] 6-105. Nerul Staion Complex (W), Fist Foo Prot S. G. Vaidya () 9820031385 9819369868, 5. Groups and Applications (Syllabus: Groups and Applications: Monoids, semigroups, Product and quotients of algebraic structures, Isomerism, homomorphism, automorphism, Normal subgroups} ‘$1 Binary Operation: A binary operation on a set A is an everywhere defined function £ AxA — A. Generally operation is defined by *, From the definition itis clear that if « is binary operation on A then a*beA Wva,bea, $2 Semigroup’ A nonempty set § together with a binary operation « is called as a semigroup if (i) « satisties closure propery ie atbeS vabes Gi) + satisfies associative propery ie. (aeb)*ena"(b*c) VaibceS ‘We denote the semigroup by (S,*) Commutative Semigroup’ A semigroup (S, *) is said 10 be commutative if * is commutative. ie ab=bea Vabes Examples}(}) (2) isa commutative semigroup, where Zi sto itegers (2) The power set PS), where $ isa set, together with operation of union is @ commutative semigroup ()@,-Visnota semigroup, where 2 isa st of integers The operation subtraction is ot associative Hdentity Element: An clement ¢ of a semigroup (S, +) is called an identity element if eranare=e VacS S&S Monoid: A non-empty set M together with a binary operation * defined on it, is called as amonoid if (Gi) + satisfies closure propery i.e. abeM Va,beM. (ii) + satisfies associative propery ie. (atb)*e=a*(b*c) Vab,ceM, (Gil) QM, +) has an identity ie there exist an element e=M such that eea=atena vaeMt i.e. A monoid is a semi group that has an identity We denote the monoid by (M,+) NIL 6.105, Nera Station Complex (W), Fist Floor Prot S.G_ Vaidya 0M 9820031384 9819365868 5.4 Group’ A a non-empty set G together with a binary operation * defined on itis called a group if (i) « satisfies closure propery ie arbeG VabeG (ii) + satisfies associative property ie. (a¥b)*e=a"(b%e) Vab,ceG. (Gi) (G, *) has an identity ie there exist an element eG such that e+a-a*e=a VacG (iv) every element in G has inverse in G ie. WaeG there exist an element a"/eG such that aea'!=a!*ae, where ¢is the identity element We denote the group by (G,*) Commutative (Abelian) Group: A group (G, *) is said to be commutative or abelian if + is commutative Ceicsme Hoey ceamtrgripcnteagnsntssoneanastundennst the group, then that group is called as cyclic group The element is called as generator of the group. IfG is a group and ais its generator then we write G= For example consider G={J, 2, 3, 4). G is a group under the binary operation of multiplication modula 5. Note that G=<2 > Note that if ‘a’ is a generator of G then ‘a’ is also generator of G roup of S SS Subsemigroup: Jet (S, +) be a semigroup. A subset T of Sis called as subsem if(T.+) itself is a semigroup. ‘Submonoid: Let (M, +) be a monoid, A subset T of M is called as submonoid of M if (T.*) itself is a monoid ‘Subgroup: Let (G, +) be a group. A subset H of G is called as subgroup of G if (,) itself is a group Necessary and Sufficient Condition for subgroup: Let (G, +) be a group. A subset H of G isa subgroup of Gif ifand only ifaeb*cH va,beH is a binary operation on the set of positive Example 5.1: Show that + defined as x*! integers. Show that * is not commutative bus is associative 112 G-10S, Nena Station Complex (M, Fst Foor ~ Prof S. G. Vaidya (M9 9820031384 9819365868 Solution: Consider two positive integers x and y. By definition x* integer. Hence «is a binary operation, =x which is a positive For commutativity: x and yex=y. Hence xeyzy*x. in general -.» is not commutative. But xe(y+z)=xey=x and (xy) Xtz=x, Hence x*(y*z)=(x+y}*z. °.+ is associative Example $.2: Let (S,*) be a commutative semigroup. Show that if x*x=x and y*y=y, then Qxty)*Oypxty. (Dec-07} Solution: Given (S,*) be a commutative semigroup. Given x*xex and y'y: Consider (x*y)*(x*y) ats Ya 0° Assocvepropey wey xrOety)ty Sis couuutative ay G9"Q"y) Associative property xty Given x*x=x and y*y-y Example 53: Let Ibe the set of integers and Za be the st of equivalence classes generated by the equivalence relation “congruent modulo m” for any positive integer m 4) Write the sets Zs dnd 25 ) Show thatthe algebraic systems (Zn) and (Zp, Xq) are monoids ©) Find the inverses of elements in Z; and Zs with respect to +5 and xa respectively Solution: ) 25 for (Zs.+3) =((0), (1), (21) Ze for Zeta) =(0}, (1, (2), (3, (4), (51) 2s for Zs, x3) =((0} (11, (21) 2 for Ze, v4) ={(0}, (1) (2), GI), (4), (51) b) We know that tm and Xm satisfy closure property and associative property. Also [0]}¢Zmis the identity element of (2m,+a). Hence Zq.+u) is a monoid. Similarly 1}eZmis the identity element of (Zn, xq). Hence (Zp, xn) is a monoid 9) For Zs.*3) . (0F'=f0), (1]"=(2}, (2)"=(1) For (Zs, x4) : (0J" does not exist. [1] *=(1], [2}'does not exist, and Gy'=B) Example 54: Determine whether the following set together with the binary operation is a semigroup, a monoid or neither. If'it is a monoid, specify the identity. If'it isa semigroup or @ ‘monoid determine whether it is commutative ') A>set ofall positive integers. axb-max{a.b) ie bigger ofaandb. —_—[May-06) i) Set $=(1,2,36,12) where ab=GCD. (4b) [Dee-03, May-07,Dec-08 May-09] it) Set $=(1.23,69,18) where ab=L.CM. (28) [Nov-06, May-1 1] iv) The set of integers Z, where atb-a+b-ab {Apr-04, Dee-107 W)The et ofeven inegecs, where arte 22 (hut oe 62 vi) Set of real numbers with ab=atb+2.~ [May-98) 13-105, Nera Sution Complex (W), Fist Floor Prof S. G Vaidya 0M) 9820031388 9819369868 ‘Solutios, i) A~set of all positive integers. arb=max(a,b} ie bigger of and b Closure Property: Since Maxa,b} is either a or b s-a.beA. Hence closure property is verified ax { {ab),¢]-Max(2,b,c} ~Max(a,(be}}= (abe Associative Propeny: Since ax(be: + is associative. (A+) isa semigroup, Existence of identity: 1A is the identity because L.a-Max{a}=a_ Vac A. (A) is a monoid Commutative Property: Since Max(a.bj=Max(b,a} we have acbb.a. Hence * is ii) Set S=(1,2,3,6,12} where a*b=G.C_D (a,b) el 2 65 pjl 23 6 2 Closure Propenty: Since all the elements of the table € S, closure property is satisfied Associative Property: Since a(bre)> axGCD(b,c}= GCD{a.b,c) [And (arb)+o = GCDfa,b}+e = GCDIabc} an(b¥c) = (asb).c + is associative (S.*) is a semigroup. Existence of Identity: From the table we observe that 12€S isthe identity (S.*} is a monoid Commutative Property: Since GCDab}= GCD{b,a} we have asb=ba Hence * is commutative il) Set Se 3,6,9,18) where aeb=L.CM (a,b) 1146-105, Nerul Staton Complex (W), First Floor ot Prot. G. Valaya (Mw) 9820031384 9819365668 +[1 23 6 9 2 7s 6 6 36 6 6 BoB 8 9 18 9 48 B18 181s is is ‘Closure Property: Since all the élements ofthe table ¢ S, closure property is satisfied Associative Property: Since a»(b*c)= aeLCM(b,c} = LCM(a,b,c) ‘And (a+b)+e = LCM{a,b}*= LOM{a,b,c} ar(bec) = (ab)ac * is associative (S,4) isa semigroup Existence of identity: From the table we observe that 1 ae Let a,b, ceG. Since Gis a group, every element has inverse inG. b" eG. Consider avb=erb Multiply both sides by b* from the right (asb)sb" = (exbyeb a(beb") = colby") Associative property ave= cre bese eG anc eG is the identity (ii) To prove the left cancel Let a, b, ceG. Since G is @ group, every element has inverse inG .a"=G 116 —_G-105, Neral Station Complex (W), Fist Floor Prof 8. G. Vaidya ov 9520031324 9819369855 Consider arb=ave Multiply both sides by 2" from the let a e(aeb) = ae(a20) (a¥*a)+b = (a"eayec Associative property etb=erc aeare eG bee 6G is the identity Example 8.6: Prove the following results for groun G. (The identity clement is unique Gi) Gi) Bach ain G has unique inverse a i) (aby'= ba? ‘Solution: (i) Let G be a group. Let ey and e; be two identity elements of G If is identity element then exes=e€1=e2 a If exis identity element then eves"e3¢;-e1 @ From (1) and (2) we get e:=e: ie. identity element is unique (i) Let G bea group. Let b and cbe two inverses of acG. If bis an inverse of a then ab=ba-e ~a If cis an inverse ofa then ac~ca=e ~@) Where eG be the identity element From (1) and (2) we get abac and ba=ca. bee by cancellation law. i. inverse of 2¢G is unique (ii) Let G bea group. Leta, beG. Consider (aby(o'a) =a(bb")a" Associative property =x(e)a" bb"=e, eG is identity “ea! ‘Associative property Similarly we can prove (b"a")(ab)re. * Hence (ab)?= ba? 117 G-105, Neat Station Complex (W), Fist Floor es fee eel oe a a Prot. S. G. Vaidya () 9820031388 819369868 Example 5.7: Let G be a group with identity e. Show that if ae for all a in G, then every ‘element is its own inverse [Nov-05] ‘Solution: Let G be a group Given ae for all ac i.e every element is its own inverse, Example 5.8: Show that if every element in a group is its own inverse, then the group must be abelian. [Dee- Let G be a group t For acG, a'=(3!, 37,3734, 97 38) Order of the subgroup generated by 3 =6 iv) Gis eyclic because G=<3> Example 5.15: Let G= (1, 2, 4,7, 8, 11, 13, 14) bea group under ‘multiplication modulo 15° 1) Find the multiplication table of G. [Dec-i0} ii) Find 24, 7 17 Find orders of subgroups generated by 2, 7, and 11 ‘Solution: (i) Multiplication table of G Binary operation * is multiplication modulo 15, ‘From the table we observe that 1=(11, 17} Order ofthe subgroup generated by 11 (iv) Since t <13> =(13°) 14> =f 4° ie. there is no generator of order 8, hence G is not cyclic : Example §.16: Let S=(x[x is a real number and x20, x=1}. Consider the following functions FSS, F127 [Nov-05] BRIE fx) Show that G={fi, fs, fi, f4 15, {6} is @ group under the operation of composition. Give the ‘multiplication tabie of G ‘Solution: (i) Multiplication table of G estar near (Closure propeny: Since all the elements in the table €G, closure property is satisfied (ii) Associative property: Since composition of functions is associative, associative propery is satisfied (Gi) Existence of identity: From the table we observe that f €G isthe identity (iv) Existence of inverse: From the table we observe that ff, BB, B16, 6 6, Bf fof 1226-105, Neral Sation Complex (W), Fist Floor a a @ » Prof SG. Vaidya Mp 9820031384 919369868 Example §.17: Let G be an abelian group with identity and let H={2 x°=e). Show that Hlis a subgroup of G. ([May-02, May-03, May-07, Dec-07] Solution: LetxyeH xe and xlex and yey Since G is abelian we have xy=yx xy""=yx Now ('}' =o" Joo) = (Yo-'s) by"Yo2)= xy) =xl0 Sry'eH. Hisa subgroup. Example 5.18: Let G be a group and let H=(x/xeG and xy=yx for all yeG), Prove that H is 2 subgroup of G LetxzeH -xy-yx for every seyxyt Similarly sy=ye for every yeG, :-z=yzy" Now consider x2"=(yxy'" yay") ye" yey > (2 yy eH, Hisa subgroup Example §.19: Find all subgroups of (Z,®) where @ is the operation addition modulo $. Justify your answer. Solution: All subgroups of (Z,) are : {0}, {0,1,2,3,4) Example 5.20: Let G be a group of integers under the operation of addition. Which of the following subsets of G are subgroups of G? a) the set of all even integers, (b) the set of all odd integers, Justify your answer. Solution: a) Let H= set ofall even integers We know, additive inverse of an even number is even and sum of two even integersis also even, Thus for abeH we have ab"'cH, 123 6-105, Nerd Station Complex (W), Fist Flot Ee Prof S. G. Vaidya (Mf) 9820031386 9819369868 Hence H is a subgroup of G b) Let K= set ofall odd integers We know, additive inverse of an odd number is odd and sum of two odd integers is even ‘Thus for a, be K we have ab cH Hence K is not a subgroup of G Example §.21: Let (G,*) be a group and H be @ non-empty subset of G. Show that (H.*) is a subgroup if for any a and b in H, ab" is also in HL [May-00] Sol (i) Let aac, :.aa"eH. ie ecH... The identity element EH. ca'eH ie a'cH Every element has inversecH. (ii) Lete, VLeta,beH bSH. -ath''y"'cH ie 1. »-Closure property is satistied (jv) Every element in His also in G. And G is a group. So associative property is satisfied by {elements of. Hence associative property is satisfied by the elements of H. Hence H is a group. But His a subset of G. «.H isa subgroup of G Example $.22: Let H and K be subgroups of a group G. Prove that HK is a subgroup of G [Dec-02] Solution: If H is a subgroups of group G, then for any a, bH, ab’! Similarly, if K is 2 subgroups of a group G, then for any a, beK, abteK. Now if a, be HK, a, beH and a, beK. -. ab"eH and ab'eK. Hence ab'e HOK. Ho isa subgroup of G 5.6 Products and Quotients In this section we obtain new semigroups from existing semigroups Theor: : 1f(S, *) and (T,+") are semigroups, then (SxT.+") is @ semigroup, where =” is defined by (s1.ts)*"(sata)(51¢52. tt). ‘Theorem $.2: If § and T are monoids with identities es and er, respectively, then, xT is a monoid with identity (es, er) Theorem §.3: Let R be congruence relation on the semigroup (S,*) Consider the relation from S/RXS/R to S/R in which the ordered pair ((a},[b) is, fora and b in S, related to (ab) @ © is a function from S/RxS/R to S/R, and as usual we denote @({a}{b)) by [a}efb). Thus (a}2(b} @) (SRG) isa semigroup. 1246-10. Nena Station Complex (W), First Floor oe uw Prof S. G. Vaidya (M0 9820031384 988369868 ‘Broof: Suppose that (a), [O)=(a', fo"). Then aRa’ and bRb’, so we must have asbRa'sb’, since R is a congruence relation. Thus [asb}=[a’*b"]; that is, @is a function ‘This means that © is abinary operation on $/R Next, we must verify that ® isan associative operation. We have IEU}eLe)-[8}EIbre] = [ar(b*c) = arbJec] by associative propery of * in § =[2*b]®[c} ~e}oRYeKI, Hence S/R is semigroup, We call S/R the quotient semigroup or factor semigroup Observe that © is «type of “quotient binary relation” on SR that is constucted from the ‘original binary relation + on S by the congruence relation R Example 523: Let Z be the set of integers, and Zq be the set of equivalences classes fenerted by the equivalence relation “congruence modulo m” for any postive integer m Znisa group with operation @ where [a] [8] = (eb) For 2a nd Zs defined according tothe above definition, write the multiplication able forthe sr0up ZZ DMtay-03) Solution: ‘The mukiplicaion table forthe group ZpxZ5. 0) 0 2) 49) G1) 2) ©) [C2 Ci) Oa) aay DOD 2 0) Gn (02) 1102) (0.0) 0.1) (12) (1,0) (1,1) 2) 1.) G2) 4.2) 9) 1) (2) GD 7D 0.2) 9 ©) @2) 9) 2) 12) 0.9) G1) 2) @o) (1) ‘Hamomerphism. Let (S, +) and (T, +") be two semigroups. An everywhere defined function £S-T is called a homomorphism from (S, +) t0 (T, +") if flarb)=f(a) *'Rb) abe Homorphism: Let (S, +) and (T, +) be two semigroups. A function f S47 is called a isomorphism from (S, +) 0 (T, #) if (5 To succuscune cutresponaence trom S$ to T and Gi flarb)=Ra) +b) Va,bes 125 6-105, Nem Station Complex (W), Fist Floor Prot S. G. Vaidya (Mp 9820031388 9819369868 Notation’ °(S, #) and (T, +") are isomorphic’ is denoted by S=T Automorphism: An isomosphism from « semigroup to itself is called an automorphism of the semigroup. Homomor Homomorphism: Let (M, *) and (M’, +”) be two monoids. An everywhere defined function FMM’ is called a homomorphism from (M, +) to (M’, +") if Rarby-ffa)*'f{b) VabeM Isomorphism: Let (M, +) and (M’, +7) be two monoids A function f M—sM" is called a isomorphism from (M, «) 10 (M’. *")f (ii) fla*b)=fla) ©°fb) VajbeM ‘Notation: °(M, *) and (M’, *") are isomorphic’ is denoted by M Automorphism: An isomorphism from a monoid to itself is called an automorphism of the monoid 9 Homomorphism. Isomorphism and Automorphism of Groups: Homomorphism. Let (G, +) and (G’, *') be two groups An everywhere defined function £G-2G is called a homomorphism from (G, #) to (G’, *') flarb)=fla) Rb) VajbeG somorphism: Let (G, *) and (G’, +") be two groups. A function fF G-+G" is called a isomorphism from (G, *) to (G', +") if () fis one-to-one correspondence from G to G" and Gi) Rarb)=fla) +b) VabeG Note: (G, +) and (G’, +") are isomorphic’ is denoted by up to itself is called an automorphism of the Automorphism: An isomorphism from a group ‘Theorem $.4: Let (S, *) and (T, *") be monoids with identity © and e', respectively. Let £ S-sT be an isomorphism. Then fle)=e' 126 108, Nenl Staion Complex (W), Fist Floor ’ . Prot. S. G. Vaidya (M) 982003184 9R19369868 Proof, Let b be any element of T. Since fis on to, there is an element ain S such that f{a)=b. Then a-ate d=fla)flare)=Hla) © e)=be"fle) Similarly, since eva, befla)=fleva)=ffe) #la)=fe)orb Thus for any beT, be be'Aefteye which means that fis an identity for T Thus since the identity is unique, it follows that fle)" ‘Theorem 5.5: Let (S, +) and (T,+') be monoids with identity e and e', respectively, Let f ST be a homomorphism, Then fle)=e" ‘Theorem 5.6: Let f be a homomorphism from a semigroup (S,*) to a semigroup (T, *"). If S* is a subsemigroup of (S,*), then F(S’)=(teT | =f{s) for some seS}, ‘The image ofS” under f, is s subsemigroup of (T, **). Proof: Ifts and t are any elements of RS’), then there exist s, and s; in $’ with ti=f{s1) and =f). Therefore, elteals)° flss)=ffsi¢s2)=flsres)=sa)*"ftsi)= "ts, Hence (T, +") is also commutative Example 5.24, Let G be a group. Show that the function £G-%G defined by homomorphism iff G is abelian [Dee-98, May-00, May-10, Dec-10J Solution: Step-1' Assume G is abelian, Prove that £G-2G defined by fla)ra? is homomorphism, LetabeG. —-.fla)=a°, {b)-b? and flab)=(ab)? by definition of Alab)=(aby* =(aby(ab) =a(bayb associativity =(a2)(bb) associativity 1276-105, Nenu Station Complex (W), Fit Floor Prof SG, Vaidya (4) 9820031384 9819369868 =fla)tb) definition off fis a homomorphism Step:2. Assume fG-G defined by f(a)=a"is homomorphism, Prove that Gis abelian LetabeG —ffa}=2*, ffb)=b? and flab)=(ab)? by definition of f fab)= flab) fis homomorphism (aby = ab definition of f (ab)(ab)=(aa}(bb) a(ba)b=a(ab)b associativity arab left and right cancellation laws Gis abelian Example 5.25: Let G be group and let a be a fixed element of G. Show that the function Su:GG defined by f,(x) = axa” for xe is an isomorphism, [Dec-02} [5] Solution: Step-1- Show that Fis 1-1 . F.Aa) = axa” Consider f(x)-f(y) forxyeG axa! G defined by {a)=a" isan isomorphism, Prove that G is abelian LetabeG. —-.fla)ma" {e)=b' and flab)=(ab)”! by definition off. fab)= f(e)R) fis homomorphism G@by"=a"b! definition of f ba Gis abelian bt reversal law of inverse Example $27: Define f(Z,+)-> (52,+) as 3)=5x, where SZ=(Sn:neZ). Verify that fis an isomorphism, (Dee-99, Dec-09] ‘Solution: Step-1: Show that Fis 1-1 Consider f(x)=fly) forxyeG Sx=5y definition of f xy fis 1-1 ‘Step-2: Show that Fis homomorphism, ForxyeG AOI=Sx ly Sy and flxty)=S(xty) “i nyew 1296-105, Neral Station Complex (W), Fis Floost Prof §. G. Vaidya (M0 9820031388 - 9819369868 =Sx45y fhetry)=fbottiy) “fis homomorphism. Since fis 1-1 and homomorphism, itis isomorphism. Example 5,28: Let G be a group of real numbers under addition, and let G’ be the group of Positive numbers under multiplication. Let £G-»G" be defined by f{x)=e". Show that fis an isomorphism from G to G’ OR [May-06, Dec-08] Show that the group G=(R,+) is isomorphic to G'=(R’,x) where R is the set of real numbers and R" is a set of positive real numbers. Solution: Step-1, Show that fis 1-1 Consider f)=Aly) forsyeG ee definition of f xy fis 1-1 Step:2: Show that fis homomorphism ForxyeG x)= fly) et and flxry)eel™ Consider Ric+y)= for xyeG Roxty)=fxdefly) fis homomorphism Since fis 1-1 and homomorphism, itis isomorphism, Example 5.29: Let G=(e, a, 0°, a, ‘mod 6). Prove that G and Zs are isomorphic. ([May-07, May-10] ‘Solution: Step-1: Show that fis 1-1 Let x= a! and y= a ‘a'} be a group under the operation of a'a'=a", where Consider fx)=f(y) for xyeG flay) ij definition of F ay fis 1-1 ‘Step-2: Show that Fis homomorphism Lets andy=% Rycu 130 G05, Nerl Station Complex (W), First Floor Prof SG. Vaidya (4) 9820031384 = 919369868 fla’) i, fa)=j and fxry)=Aa's'y Consider ftxty)=f(a’) = ff!) where itj=r(mod 6) aye) £ fhoxy)=fx) fly) -fis homomorphism. Since fis 1-1 and homomorphism, it is isomorphism, Example 5.30: Let T be set of even integers. Show that the semigroups (Z,+) and (T+) are isomorphic, where Z isa set of integers [May-05, Dec-07, May-11] ‘Solution: We show t | Define f(Z.+)~ Show that fis 1-] Consider (x)=fly) 2x=2y is one to one onto sy field . ‘Show that Fis onto yr2x xeyi2. when y is even for every yeT there exists xeZ Fis onto, Example §.31: For the set A=(a,b.c} give all the permutations of A. Show that the set of all Permutations of A is a group under the composition operation Sulton: (5B): $e oo percuaons of fabe be ‘a bc) i a los of coy cba) e{2? 9, be) (abe liters ca eta Ss=(f, fi, 6, 6, 6, 6} Lets prop ik conpostin ube Gal Tae mn conan wn rue: l pean Prof §G. Vaidya (4) 9820031384 om 936 9868 i) Closure Property: Since all the elements in the composition table €Ss, closure propery is satlfied ii) Associative Property: Since composition of permutations is associative, associative property is satisfied iif) Existance of ldemtty: From the table we find that fs the ide iv) Existance of Inverse: from the composition table itis clear that fy =fa, fr Every element has inverse in Ss Hence Ss is a group, s10¢ Norm eft Coset: Let (H+) be subgroup of (G..). For any a€G, the set of aH defined by h/heH) is called the left coset of H in G determined by the element acG. The element ais called the representative element of the let coset ald Let (Hi) be @ subgroup of (G,.), For any a€G, the set of Ha defined by H=[he, /heH) Right Cos. clement ais called is called the right coset of H in G determined by the element acG. The the representative element of the right coset Ha ‘Theorem 5,7: Let (H.) be a subgroup of (G.). The set of left cosets of H in G form a Partition of G, Every element of G belongs to one and only one left coset of H in G, ‘Theorem 5.8: The order of a subgroup of a finite group divides the order of the group, Corollary : If (G,.) is a finite group of order n, then for any acG, we must have ae, where is the identity ofthe group, (6-105, Nera Siation Complex (W). Firs Foor - Prot SG. Valiya @ 9220031384 9819369868 Normal Subgroup: A subgroup (H,.) of (G,) is called a normal subgroup if for any acG, ali-Ha Example $32: Determine all the proper subgroups of symmetric group (Ss,0). Which of these subgroups are normal? Solution: S=(1, 2, 3). Ss= Set of all permutations of S Sif fff, ff} where 2 di e(i2 o(! 2 2) 274°3, From the table itis clear that {f,fi},{ f, &.).(f. 6} and (ff, 3) are subgroups of (Ss), The left cosets of (fi, fi) are (6, fi}, (ff), (6, £4). While the right cosets of (f, fi} are( ffi) (6, £3).(6, 6). Hence (ff) is not anormal subgroup Similarly we can show that (f,) and (f, 8 are not normal subgroups, (On the other hand, the left and right cosets of {f, ff) are (f, ff) and {f, ff) Hence {f, ff} is anormal subgroup. Example §.33:_ Let $={1,2,3), Let GS, he rhe ~-rnn ofall permutations of elements of S, under the operation of composition of permutations 23) >). reste Lee bentzoe tee matin (2 3 2 ‘ef cosets of H in G. Is H a normal subgroup? Explain your notion of composition clearly (Dec-02, Nov-06, May-08] 1336-105, Nenu Suton Complex (W), Fist Floor Prof SG. Vaya 0) 9820031388 SR19369868 Exercise (1) Determine whethe? the following set together with the binary operation is @ semigroup, a monoid or neither If it is @ monoid, specify the identity. If itis a semigroup or « monoid determine whether itis commutative (® Qthe set of al rational numbers with the binary operation of addition (@—Theset of all mxn matrices under the operation of addition, (@) Determine whether the set Q, the set ofall rational numbers with the binary operation of addition is a group. If tis a group, determine if iis abelian; specify the identity and the inverse of a general element @) 1G is set ofall non-zero real numbers and a show that (G,*) isan abelian group {May-03} (4) Let Z, denote the set of integers (0, 1, ., n+l), Let @ be binary operation on Zy such ‘that ab = the remainder of ab when divided by n. () Construct the table for he operation @ on n=4, (Gi) Show that (Zq, ©) @ semi-group for any n. il) Is Zp, ®)a group for any n? Justify your answer. (5) LeG bea set of integers between | and 15 which are co-prime to 5, Find the multiplication table of G.Find?', 7, 11" IsGeyclie? [May-05) (6) Prove thet ae when G is finite group, and e is its identity element (7) Let G bea group with identity & Show that ifa°e for all ain G, then G is abelian [Mtay-05) (8) Determine whther the fllosng sets along with the binary operation, form a group. it i group, sate the deny, and the inverse ofan element a. Ii is mot a group: stat the reson why? foe} seis rs that of union + ofall subsets of $ where § is a non-empty set. The operation is (i) Set oF all non-zero real numbers, under the operation of multiplication (9) Let! be» subgroup () Lef coset of Hin G. (i) Right coset of Hin G 2 group G. Define the followit [02 136 G-105, Nerl Station Complex (W), Fis Floor o Prot $C. Vaidya (9820031384 sa1s36s068 [Syllabus: Codes and group codes | 6.1 Introductio ‘A message isthe basic unit of information. It sa finite sequence of characters from a finite alphabets. We shall choose the set of finite alphabets as B={0,1}. Every character or symbol that we want to transmit is now represented as sequence of m elements from B, Our message is also called as word. It isa sequence of m 0°s and I's, ‘The set B isa group under binary operation © ‘The process of sending a word from one point to another point is shown below ( word x € m) meen? hase 5 <) rremeaine received We send a word xm and a one-to-one function ¢:B® B®, The function eis called (m,n) encoding function, and we view it as a means of representing every word in B™ as a word in B If beB", then e(b) is called the code word representing b. The additional 0°s and 1's cain provide the means to detect or correct errors produced in the transmission channel. ‘We now transmit code words by means of a transmission channel. Then each code word x=e(b) is received as the word x.¢B", This situation is illustrated in the following figure, se *") (encoded word: = (6) « B°)— meee (eS ea } If transmission channel is noiseless then x,=x for all x B" be an (im,n) encoding function, We say that e detects k or fewer errors if whenever x-e(b) is transmitted with k or fewer errors, then x; is not a code word (thus x. could not be x and therefore could not have been correctly transmitted), For x€B", the ‘number of 1's in x is called weight of x and is denoted by fx Weight of a code word: It is the number of 1's present in the given code word Hamming distance between two code words: Let xxix: Xm and y=yiys. Ym be two code swords. The Hamming distance between them, 5(x,y), is the number of occurrences such that for i=1,m, _ Exfmple 6.1: Find the weights of the following [Apr-04, May-06) (2) 010000 (6) x=11100(€) x=00000 (4) x=11111 (@) x=01001 _(f)x=1 1000 Sol (@) ie! = | 010000) @ p= y= 1 (b) |x] = 11100} =3 (©) [xj = 00000) = 0 © bi 2 (Os) =|11000]=2 samole 6.2: Find the Hamming distance between thecode words: [Apr-04, May-08] (a) x=-016000 y=009}01 —(b) x-00]190 y=010110 Solution: (a) (x3) = fx€y1=| 010000 © 000101) =[019103] = goods (©) (xy) = @y{ =| 001100 © 010119] = o110191=3 ° oor an we yo Example 6,3: Let d be the (4,3) decoding function defined by dBY9B? Ifyyiy.. ei aty)yiy2-¥m Determine dy} forthe word yinB*(a) y=0110(&) y=1011 {Nov-06,May.09] ‘Solution: (a) d(y)=011 (b) €(9)=101 Example 6.4: Let d:B°-»B" be a decoding function defined by for y-yiya.Ye Then d(y)=2122 where Zit] if {ysyeayiea} has at lear* two I's © if yayiryiea} has less than two T's 138 —_G105. Nena Station Complex(W) Fist Floor uv Prof SG. Vaidya (up 982008138 = sensisoees Determine d(y) forthe word y in B* (@ yernon () 010100 Solution: (a) d(y)=11 (b) dG)-01 nA 4 Example 6.5: The following encoding function e B"-> B™"" i called the parity (mm+1) - check code. IFb=bib;...yeB", define e(b)=byba,.. bm where gol, 012 1-0 fis even go>, el 1 iffblis ode asp Some Find e(b) if(@ b=01010 Gi) b-01110 Solution: (a) eb)=QJOLVO(b) etb)= ouyjoL Ne Consider the following (m,3m) encoding function e B"> B™ If br bweB™, define ‘ [May-10} e(b)=e(bibz... ba}= (Dibz...bm) (bib2... brs) (bib2...bm) - write code words of all 3 bits B?={000, 001, 010, 011, 100, 101, 110, 111} +2(000) = 000 000 000 ‘e{001) = 001 001 001 (010) = 010.030 010 e01i)= 011 011 O11 (100) = 100 100 100 e(101)= 101 101 101 (110) = 120110 110 equny= tat eB oB is 2 Grown Cade: An (aa) encoding ‘group code if e(8"}{e(6)/ beB™) is a subgroup of BY 6.3 Additional Results from Boolean Mi (a) Mod:2 Addition: Consider the set B with +. Now let D=[dj] and ices. We denote the mod-2 sum D@E as the mxn Boolean matrix F=[f;) =[e;) be men Boolean where firdrey, lsism Isjsn | Here + is addition in B. For example Prot 8. G. Vaidya (6) 9820031384 9819369868 10 on 10 (0) Mod-2 Product: D*E fii ol, fod Theorem: Let D and E be mxp Boolean matrices, and F be a pxn Boolean matrix. Then (DEE)F=D+F)HE+F) ‘That is distributive property holds for @ and * Let m and n be non-negative integers with mén, r=nem, and let H be an nvr Boolean matrix. Then the function fi:B"-+B' defined by faQXeH, xB", we often need to determine an (n,m) decoding function d:B"B" associated with e We now discuss a method, caled the maximum ene. Since B™ has 2” likelinood techniques, for derermining a decoding function d for a gi elements, there are 2” code words in B". We first list the code words in a fixed order. Iftte received is sj, we compute Sal"s) for 1 <1 < 2" and choose the first code word. say it is, such that min (5(x"x)} = 8(9°%x9 ‘That is, x is a code word that is closest to x, and the first in the list. Ix e(b), we define the maximum likelihood decoding function d associated with e by Observe that d depends on the particular order in which the code words in e(B") are listed. If the code words are listed in a different order, we may obtain a different likelihood decoding function d associated with e \B defined by ‘e(000) = 0000000 (001) = 0010110 (010) = 0101000 (011) = 0111110 (100) = 1000101 ‘e(101) = 1010011 e(110)= 1101101 e(111)= 1111011 isa group code. Solution: 000000 (oot0110) o101000 o111140 toWi01 1010011 1101101 1111011 oo10110 01009 ono 010110 0000990 6111110 0101000 1010011 Jeo0101 HI011 110110 01099 0111110 bo¢0%— OO10110 1101101 1111011 1000101 roL002t too010h 010011 Ligito. 144 10110 O101000 0141110 1010011 1000101 LL11011 1101101. ao10110 110 101100 101101 L111011 1000101 1010011 0101000 O11 1810 oor0110 tio ‘099000 010011 tro101 nui Since closure property is satisfied, itis « group code Exgfnple 6.9: Consider the (2,4) encoding function e as follows, F 00) = 0000 (01)= 0110 10) et) = 1100 How many errors will e detect? [M 1-06, Dec-09, May=11) Solution’ © | 0000 ono out 100 Minimum distance between distinct pars of €= 2. “.kt+1=2 2k The encoding function e can detect | or fewer errors Example 6.10: Consider the (2,6) encoding function e(00)=000000, e(10)=101010, (01)=011110, e(11)=111000, Find the minimum distance of e How many errors will © deen? (Dec-07) 10. ath it st Cams Fe Ph raf 8. G. Vaidya (M) 9820031384 a 9819369868 e 00.000 101010 oll 011.200 00.000 7 woroi0 oN Ott 000 101 010 = 1010110010 ou no = 100 110 111 000 — Minimum distance =3 ke ke The encoding function can detect 2 or fewer errors Example 6.11; Consider the (3,8) encoding function e:B?->B" defined by (000) ~ 00000000 (100) ~ 10100100 (001) = 10111000 ;o001001 010) = 60101101 (110) = 000111100 e(011) = 10010101 e111) = 00110001 How many errors will e detect? (Dee-08, Dec-10} Solution Gon _seco1to1 enantio. onnLLiMn 1 S001 oxen rolonien 10111000 10001001 wor01101 90011190 Lov49101 o0110001 o100t00 <= b00L1190. 00101101 10001001 10111900 90110001 s0010101, wi - 01100001 16010101 10100100 00101101 10001001 0001001 <= 10100100 19010101 9011100 Lox11000 o10i101 = 00110001 10111000 011100 00100 10001001 09101108 10010101 —~ —ro1o100 0110001 Minimum distance between pairs of e = The encoding fyncton e can detect 2 o fewer erors he te 1 1%9 hi ri » ° 1 ° 28° (May-07 Dee-10) ©}. Determine the group code ey:B* at a 6.12: Let H = | ’ l Solution: We have B={00,01,10,11}. Then €(Q0)-00r%2%9 where Me oP tart Ayrskgn 9 wean oe 201.0) 44! GH0S, Nerul Station Complex(W), First Floor Prof S.G. Vaidya (Mp 9820031384 9819369868 x1 0140.0=0 mr 0.1401=0 mr 0080.10 = -f00)-00000, Now r (O10 1xpaxs where 1H1.0~0 mr 0.1HLI= 9" 0041 e(01)-01011 Next ©(10)10x1x2%5 114001 w=1.040.1-0 , €(10)=10110 (11101 a bey [May-03, Dec-08, Dec-09} ihe 10d Ha op #H ° ° 1 10 o1 oo be a parity check matrix. Determine the (3,6) group code ey:B?->B* Solution: We want to find e(000), e(001), e(010), e(011), e(100), e(101), e(110), e111) «(000}-000000 «(100)=100100 ¢4001)-001111 (101101011 (010010011 (119110111 . e(111)=111000 __Example 6.14: Consider the group code defined by e B*sB’ such that 00} = 00000 (01) = 01110 e(10) = 10101 (11) = 11011 145 _G-105, Nenu Station Complex(W), Fist FLOOE Prof S.G.Vaidva (M) 9820031384 s 9819369868 Decode the following words relative to maximum likelihood decoding function @ ine () 10011 [Dee-05} © 10100 Solution: a) x=1110 ‘Compute (x'",x,) =[00000811110|=|11110)=4 (x x0) =/011 1011 1 10}=|10000}=1 8x") =|10101611110/=]0101 1/=3 (x, x,) =|1101111110}=[00101)=2 min {8(x°.x)} = 1-560.) €(01)=01110 is the code word closest to x= 11110. The maximum likelihood decoding fu by xe10011 Compute &{x'",x) “400000610011 [11101 )=4 {x \x) “0111081001 1-001 10)-2 8(xx) “10101011 110}=fo1011 3 &(x"°,x) =[110111001 -[01000)=1 min {6(s.x)) = 1=8(4.x) €(11)=11011 isthe code word closest to x=10011 tion d associated with eis defined by 4 ‘The maximum likelihood decoding function d associated with eis defined by d(x)=11 «) x=10100 Compute —_&(x",x) =|00000@10100}+|10100)=2 8(x.x) 011 10810100-|1 1010}-3 (xx) ={10101610100/=]0000: &(x".x) =[11011@10100)-)011 11/4 sin {6¢x°.x)} = 1=5( x) @(10)=10101 isthe code word closest to x=10100 ‘The maximum likelihood decoding function d associated with e is defined by ox)=10 146 G-108, Nerl Station Commples(W), Fest Floor Prof. SG. Vaidya (vp 9820031384 818369865, Example 6.18: Show that (2,5) encoding function e: B°»B* defined by e(00)-00000, ©(01)-01110, e(10)=10101, e(11}=11011 is a group code. Cansider this group code and Aecode the following words relative to maximum likelihood decoding function. (10 — () 10011 [Apr-04, May-06, Dec-09, Dec-10, May-11] Solution: Group Code: ee xs eG ® ollie 0101 Holt Bo000 Toro Torr onto Hott iio 0101 00000 orto Hot 0111090000 Since closure propery is satisfied, itis @ group code. Now let x =00000, x=91110, x®=10101, x=11011 (@)x=11110 86" Bx x) 8x Bon) = fx"@ x)= | 11011 © 1110] =Joo101] = 2 Maximum likelitood decoding function d(x) = 01 (b) x= 10011 86x.) = PB x |= | 00000 6 10011] = 10011) = 3 5.x) = k® x |=] 01110 10011] =|11101]= 4 59°.x) = ! x| =] 10101 & 10011) = 00110] = 2 B(x.) = 6 ® x, |=] 11011 @ 10011] = 01000] = 1 ‘Maximum likelihood decodi function d(x) = 11 v4 oid EMEILEIS Coniterpiy otek matic ony =} 0 0] 010 Determine the group code ey’B™>B* (Apr-04, May-07,.May-09) 1476-105, Nem Staion Complex(W), First Leer Prof. $.G. Vaidya (M) 9820031388 Decode the following words relative to a maximum likelihood decoding function associated with ex 01110, 11101, 00001, 11000, Solution: By={00,01,10,11) Oxixeks where xi=0.140.00, x°0.140.1=0, 040 1 (00) where x:*0.1+1,00, LIL ex(01)-O ax .ex(00)-00000 X9°0.041. 151 -.en(01)=01011 ex(IO)=10xrxx3 where xi=1.1+0.0-1, xeLHO1=1 x=] 040 1-0)» en(01)-10110 SELF 126, OHLI=T ey(0 Desired group code = (00000, 01011, 10110, 11101} () xeo1110 (Lu) = PLO xe (00000 6 01110) = [01110] = 101 xi] =] 01011 © 01110 1119] = 11000} (07a) = be! 8x) = KO x |=| 101106 (0.x) = fx? xe [= | 11101 6 01110] = [10011 01 Maximum likelihood decoding function dx) Q)x=I1101 (ex) = PB x |= | 000006 11101) = [11101] =4 7.x) = MO x, B(x.) = [x°@ xy = | 10101 @ 11101] =\01011) =3 8(x!,x) = fx'® xy |= | 11011 @ 11101) = 00000] = 0 =i 01110 11101] =|10110}=3, Maximum likelihood decoding function dx) (ax) = pS x | = | 00000 6 00001) = 00001) = 1 (3.x) = x x |=} 01011 © 00001} » 01010) 5x, @ x | =| 10110@ 00001] = [10111] =4 148 —_G-10S, Nenu Station Complen(W rot 8G. Vaidya (0 9820031388 5819365868 Maximum likelihood decoding function d(x; (3) x=11000 8(x"?.x) = fx" x, =| 00000 @ 11000) = [1 1000] = 2 ‘i » (xx) = @ xe | = | 01110 & 11000] = [10011] =3 x) = KO x= | 10101 © 11000) =|01101] =3 Bex) = WS x: = | 11011 @ 11000) = [10000] = 1 ‘Maximum likelihood decoding Function df) = 11 7 jor Example 6.17 Let =| it | be a parity check matrix. Decode 0110 relative toa maximum 1 : with e [Dec-04,May.09} Solution: 61:98 Br=(00,01,10,11) en(00)=00xix where x1=0.140.0 x:=0,140.1-0, ex(O1)=O1 xe; where »1=0.1-41.0=9, x=0.141. 161, €x(01)-0101 s eH(10)=10spx3 ex(01)=1011 ex(1 1) Mies where x1=1.1+1.0=1 xemLI+L1=0, ex(01)=1110 Let x!'=0000, x=0103, x2 Let x=0110 11,=1110 86x.m) = YG x | =| 0000 © 01104 = 110] =2 8x) = © x,| =| 0101 & 0110) =o011|=2 BM) = k@ x/|=| 1011.8 0110) =|1101]=3 Bex) = MB x=] 11106 0110) (000) = 1 Min 56x!) = 8(¢.n)) and e(11)= x. d(x)=11 Example 618 Consider the (2,5) group encoding function defined by , £(00)=00000, e(01)=01101, e(10)=10011, e(11)=11110 1496-105, Nenu Suion Compiex(W), Firs Floot Prof. G. Vaidya (0 9820031384 2 5815369868 en(001)"001xix2 where xj=0.140.0+1.1=1, $0,140,141 x(001)=00110 ex(010)-010six; where x1=0.141.040.10, 2=0.141.1+0.0=1, e4(010)-01001 en(O11)-O1 Lxixe where x)=0,141.0+1.1=1, 1+1.1+1.0=1, en(OL1)=O1111 en(100)=100% where xi=1.140.0+0.1=1, SL140140061, -.¢4(100)=10011 en(ION=10lxix: where xy=1.140.041.1 RL 10141061, .¢4(001)=10101 ex(110}*110xpx where x/=1.1+1.040.1=1, NFLIFLI+1060, — --ey(110)=11010 ex(1I)=11txx2 where x=1.1+1,04+1 1-0, NeLIFL1+1.00, — -sex(111)=11100 Let x=00000, x”=00110, "=01001, x=01111, x =10011, x =10101 x #11010, x=11100, GQ) Let #00111 B(e.x) = f® xe] =] 00111]=3, 8x7.x)= KMS, 1 8.x) = KPO x, |=] 01110] = 3, 3x.) = x | = | 01000) = 1 (xx) = KD x, 1 10100|= 2, &(x")x.) = eS x.) =| 10010] =2 Bx) = Bm] =| 1011 =4, 568x) = MO x| =| LOT] <4 Min 39) = 868%) and ef0019* x) du )=001 @)Let xeto1n1 B(x.) = [x=] 10111] =4, 866 x) = xO x] =| 10001] =2 8°.) = 1 x5|=| 11110] =4, 5(0%x) = pW x |= | 11000 B(x.) = [x x4 | = | 00100] = 1, B(x) = OB x |= 00010) = 1 8.x) = x=] 01101] =3, 8x) = [xB x, |= | 01011 Min 8(x!°,x,) = 8(x),x,) and e(100)= x) d(x)=100. @) Let x=11001 eM Maat =yruy=> iw USL 6-05, Ne Station Compes(W), Fist Floor 2 Eee Prof § G. Vaidya (M) 9820031384 = oe t936s4ce BER" .x) = PD x |= | 10000] = 1, 66x) = x xe |=] 1011 B(x.a}) = 1D x |= | 01010} = 2, B(x) = 1S x, | = | 01100) Bx) = OB xc =| 00011|= 2, B(x) = fx xe] = | 00101] = Min B(x (x) and e(010)= x" .d(x)-010, fi 1 oj fio. tamale 620 Ls | | '| ve a pry tek mari Demin he coneonting O10 oma iy Bede eal tats emcee ‘Solution: Given H is 4 parity check matrix of (3,6) group code. ex:8'+B° B?={000, 001, 010, 011, 100, 101, 110, 111} ¢x(090)=000000, en(001)=001011, ey(010)=010101, _ex(011)=011111 ex(100)=100110, en(101)=101110, —ex(110)= 110011, ex(111}=11 1000 (i) Min distance ofa group code = min weight of non-zero code Word = 3 weiss k=? The group code can detect at the most 2 or fewer errors. (li) Maximum likelihood decoding procedure: Let ex(000)-x', en(001)= x, en(010)= x, en(011)= x", en(100)= x, en(101}=x®, en(110)= x, ey(111}= x and let « be transmitted codeword Find (xx), take minimum, IfMin 86x) d(xa)=b where entb) ‘8(x"°.x.)_ then maximum likelihood decoding function d ean be defined as Tf two or more x" have the same minimum value then we select the x! whichever comes first im the list and define the decoding function accordingly Example 6.21: Consider (3,6) encoding function e as follows: [May-07, Dec-07, Dec-09] (000) = 900000, &(001)= 000110, €(010)= 010010, e(011)=010100 e(100)= 100101, ef 101)= 100011, e(110)= 110111, e(111) = t10001 i) Show that the encoding function e is a group code ii) Decode the following words with maximum likehood technique: (101101 (ON! (eyo Prof S. G. Vaidya 0M) 9920031384 9819369868, ® 900000 090110 010010 010100 100101 100011 110111 110001 “aso oT Ta Se a oe 00110 000110 000000 o10100 010010 100011 100101 110001 110111 10010 Jo10010 010100 900000 o0110 110111 110001 100101 100011 10100 010100 010010 0001 10 000000 110001 110111 100011 100101 100101 /100101 100011 110111 110001 00000 000110 010010 010100 Fo0011 100011 100101 110001 110111 000110 000000 010100 010010 Hot11 |110111 £10001 100101 100011 610010 010100 G00000 0001 10 1000) {119001 110111 100011 100101 Gtos°2 010010 000110 000000 Since the closure property is satisfied, itis @ group code. (i) Let = 00000, 00110, x= 010010, x= 010100, x= 100101, x= 100011, 8 = 110111, <= 116001 (@)Let x=101101 8(20.x) = IMB x)= | 0000008 101101 |= | 101101 |= 4 Bx) = KO, Bx x) = OD x, 00110 101101 |= | 101011 |=4 0100106 101101 |= | 111111 =6 8¢¥%x) = be" x5|=|010100 101101 |= | 111001 |= 4 B(x) = [8 x:| =| 100101 @ 101101 |= | 001000) = 1 8.x) = N® x)= | 100011 & 101101 |= |001110|=3 8x) = WS x] =| 110111 101101 |= | 011010) =3 8x" 110001 © 101101 |= | 0111000 | = 3 Min 8¢e.x) = 6(<2x). Thus x is the code word tha is closest to x, and e(100)= x ed with eby dx)=100, REx, We define maximum likelihood function d ass () Let 011011 BOM .xy= PD x, 000000 o11011 j= | o11011|=4 (xx) = k@ x, |= | 000110 @ 011011 j= | o11101 5(x).x) = x°@ x| = | 010010. 011011 |= | 001001 5ex!.xi) = jx @ x |= | 0101006 o11011 [= |o01111 86x .xy) = fe x, |= | 100101 @ O11011 |= | 111110) 8.x) = KB x, |=| 100011 © 11011 |= | 111000) 5ex.x.) = Bx =| 110111 @ 01101) |= | 101001 =3, 153 G05, Nenl Station Complex(W), First FA00F : —————E———ee Prof §. G. Vaidya (4) 9820031384 = 9819369868, 5E3.x) = [8@ x |=] 110001 6 011011 |= | 101010 |=3 Min 8(x°.x) = &(x) Thus x°” is the code word that is closest to x, and e(010)= x We define maximum likelihood function d associated with e by d(x)=010. (©) Let w@011110 5x()x) = fN@ x | = | 000000 © 011110 |= | 011110 8(x x)= I x |=] 0001106 011110 |= {011000 |= 2 (B* 'B'={000, 001, 010, 011, 100, 101, 110, 111} (000) = 000000=x" e001) = 001011=x (010) = 010110 x” en(011)=O11101=x ——ey(100) = 100100= x ey(101) = 101111 x ex(110)= 110010 x en(111) = 111001= x" @ Let x-011001 (x4) = (xB xy] =| 011001 |= 3, 8x) = kB x,|=| 010010 |= 2 5x) = KD xi] =] ONT |= 4 Bex! x9= W%@ wi = | 009100) = 154 G-105. Nerul Station Complex(W), First Floor | |

Você também pode gostar